Variables aléatoires discrètes

VAD

(ENS PC) Correction

Une suite de VA $(X_n)$ d'un espace probabilité $(\Omega,\mathcal{F},P)$, prend ses valeurs dans $\{0,\frac{1}{n},\cdots,\frac{n-1}{n}\}$, et vérifie: $$\forall k\in \inter{0,n-1},\quad P(X_n=\frac{k}{n})=\alpha_n\left(\ee^{k/n}-1\right),\quad\text{ avec }\alpha_n=\dfrac{1}{\sum_{k=0}^{n-1}(\ee^{k/n}-1)}.$$

  1. Trouver un équivalent de $\alpha_n$ lorsque $n$ tend vers $\infty$.
  2. Pour $n\geq 2$, on note $F_n$ la fonction de réparation de $X_n$.
    1. Calculer $F_n(x)$ pour tout $x\in \R$.
    2. Montrer que $F_n\overset{CS}{\tendversN}{\,F}$, continue sur $\R$. Donner l'expression de $f$.

Correction

  1. On note $f$ la fonction définie par $f(x)=\ee^x-1$, $f\in \CC^\infty(\R,\R)$. En utilisant la somme de Riemann, on a $$\dfrac{1}{n}\dsum_{k=0}^{n-1}\left(\ee^{k/n}-1\right)\tendversN\, \int_0^1f(x)\ud x =\ee -2.$$ Donc $\alpha_n \simeq \dfrac{1}{n}\dfrac{1}{\ee-2}$.
    1. Soit $n\in \N,\, n\geq 2$, Soit $x\in \R$, si $x\leq 0$ alors $F_n(x)=0$, si $x\geq 1$ alors $F_n(x)=1$, pour $x\in ]0,1[$, il existe $k\in \inter{0,n-1}$ tel que $\dfrac{k}{n}\leq x< \dfrac{k+1}{n}$ ($k=\lfloor nx\rfloor$). Dans ce cas $F_n(x)=\alpha_n \dsum_{j=0}^k \left(\ee^{j/n}-1\right)$. Ainsi, $$ F_n(x)=\left\{\begin{array}{lcl} 0&\text{si}& x\leq 0\\ \dfrac{n}{\alpha_n}\dsum_{j=0}^{\lfloor nx\rfloor} \dfrac{f(j/n)}{n}&\text{si}& x\in ]0,1[\\ 1&\text{si}& x\geq 1 \end{array} \right.$$
    2. Soient $n\geq 2$ et $x\in ]0,1[$, on écrit $x = \dfrac{\lfloor nx \rfloor}{n}+\eta$ avec $\eta< \dfrac{1}{n}$, puis $$\int_0^xf(t)\ud t =\int_0^{\frac{\lfloor nx \rfloor}{n}}f(t)\ud t +\int_{\frac{\lfloor nx \rfloor}{n}} ^x f(t)\ud t =I_1+I_2.$$ Ensuite, $$\begin{array}{lcl} I_1 - \dsum_{j=0}^{\lfloor nx\rfloor} \dfrac{f(j/n)}{n}&=& \dsum_{j=0}^{\lfloor nx\rfloor -1 }\int_{\frac{j}{n}}^{\frac{j+1}{n}} \left(f(t)-f(j/n)\right)\ud t\\ &&\\ &\leq & \dsum_{j=0}^{\lfloor nx\rfloor -1 }\int_{\frac{j}{n}}^{\frac{j+1}{n}} \abs{f(t)-f(j/n)}\ud t\leq \norme{f'}_\infty^{[0,1]} \dsum_{j=0}^{\lfloor nx\rfloor -1 }\int_{\frac{j}{n}}^{\frac{j+1}{n}} (t-\frac{j}{n})\ud t\\ &&\\ &=&\norme{f'}_\infty^{[0,1]} \dsum_{j=0}^{\lfloor nx\rfloor -1 }\dfrac{1}{2n^2}\leq \dfrac{\norme{f'}_\infty^{[0,1]}}{2n} \end{array} $$ Et, $$I_2\leq \int_{\frac{\lfloor nx \rfloor}{n}}^x\abs{f(t)}\ud t\leq \norme{f}_\infty^{[0,1]} \dfrac{1}{n}.$$ Le même raisonnement montre aussi que $\abs{\dfrac{n}{\alpha_n}-\dfrac{1}{\dsp\int_0^1f(t)\ud t}}\leq \dfrac{K}{n}$ avec $K\in \R_+^*$.
      On en déduit, $$\forall x\in ]0,1[,\,\, F_n(x)\tendversN\, \dfrac{1}{\ee-2}\int_0^x f(t)\ud t =\dfrac{\ee^x-x-1}{\ee-2}.$$ On en déduit finalement que $$F_n \overset{C\,S}{\tendversN\,}F,\,\text{ avec } F(x)=\left\{\begin{array}{lcl} 0&\text{si}& x\leq 0\\ \dfrac{\ee^x-x-1}{\ee-2}&\text{si}& x\in ]0,1[\\ 1&\text{si}& x\geq 1 \end{array} \right.$$



($\star\star$) Correction

Dans une urne figurent $N$ boules numérotées de 1 à $N$ (avec $N \geq 2$). Dans celle-ci on opère des tirages successifs (avec remise) jusqu'à l'obtention d'une série de $k$ boules consécutives identiques ($k \geq 2$). On note $T$ la variable aléatoire déterminant le nombre de tirages opérés à l'arrêt du processus.

  1. Montrer qu'il est presque sûr que ce processus s'arrête.
  2. Déterminer $P(T = k)$ et $P(T = k + 1)$.
  3. Soit $n \geq 1$, établir $P(T = n + k) = \frac{N - 1}{N^k}P(T > n)$
  4. En déduire que la variable $T$ admet une espérance et déterminer celle-ci.

Correction

  1. Pour $n\geq 2$, on note $S_n$ l'évènement : "La boule obtenu au tirage $n$ est idéntique au tirage $n-1$".

    On note dans la suite l'évènement $\text{\textcolor{red}{SUCCES}}$ :="On a obtenu une série de $k$ boules consécutives identiques".

    On note également pour $n\in \N^*$, l'évènement $A_n$: "On a obtenu un $\text{\textcolor{red}{SUCCES}}$ au cours de $n$ premières tirages", ainsi que $u_n=P(A_n)$.

    Il est clair que $A_n\subset A_{n+1}$. On a aussi (les tirages sont supposées indépedantes) $$\forall n\geq 2,\quad P(S_n)=\dfrac{1}{N},\quad P\left(\overline{S_n}\right)=\dfrac{N-1}{N}.$$ Les évènements $A_1,\cdots, A_{k-1}$ sont impossibles, et on a $$ A_k =S_2\cap S_3\cap S_4\cdots \cap S_{k-1}\cap S_k.$$ Donc $$u_1=u_2=\cdots=u_{k-1}=0 \text{ et } u_k=\dfrac{1}{N^{k-1}}.$$ L'évènement $A_{n+1}\setminus A_n$ signifie qu'on a obtenu un $\text{\textcolor{red}{SUCCES}}$ pour la première fois au range $n+1$, i.e. $$A_{n+1}\setminus A_n = S_{n+1}\cap S_n \cdots \cap S_{n+1-(k-1)} \cap \overline{ S_{n+1-k}}\cap \overline{A_{n-k}}.$$ En effet, les $k$ tirages :$n+1-k,\cdots,n+1$ sont identiques, et le triage $n+1-k$ est différent de tirage $n-k$ On en déduit alors, que : $$\forall n\geq k,\quad u_{n+1}-u_n=(1-u_{n-k})\dfrac{1}{N^{k-1}}\dfrac{N-1}{N}\geq 0$$ La suite $(u_n)$ est croissante, majorée (par $1$), donc convergente. On vérifie facielement que $$u_n\tendversN\,1.$$ La suite $(A_n)$ est une suite croissante, d'après le théorème continuité monotone $$P\left(\dsp \bigcup_{n\in \N^* }A_n\right) =\lim_{n\to \infty}P(A_n)=\lim_{n\to \infty} u_n=1.$$ L'évènement : "Le processus ne s'arrête pas" se traduit par $\overline{\dsp \bigcup_{n\in \N^* }A_n}$, or $$P\left(\overline{\dsp \bigcup_{n\in \N^* }A_n}\right)=1- P\left(\dsp \bigcup_{n\in \N^* }A_n\right)=0.$$ Ainsi $T$ est finie et bien définie.

  2. L'évènement $(T=k)$ peut s'écrire sous la forme: $$(T=k)=S_2\cap S_3\cap \cdots \cap S_k\text{ donc } P(T=k)=\dfrac{1}{N^{k-1}}.$$ L'évènement $(T=k+1)$ peut s'écrire : $$(T=k+1)=\overline{S_2}\cap S_3\cap\cdots \cap S_k\cap S_{k+1}$$ ce qui donne $P(T=k+1)=\dfrac{N-1}{N}\dfrac{1}{N^{k-1}}=\dfrac{N-1}{N^k}$.
  3. Soit $n\geq 1$, $(T=n+k)$ correspond à $k$ tirage de boule identiques entre le tirage $n+1$ et $n+k$, à condition que $(n+1)$ème tirage soit différent du $n$ème tirage et également on n'a pas eu une série de $k$ tirage identiques entre le premier et le $n$ème. Ce qui donne $$(T=n+k)=\overline{A_n}\cap \overline{S_{n+1}}\cap S_{n+2}\cap \cdots\cap S_{n+k} \Longrightarrow P(T=n+k)=P(T>n)\dfrac{N-1}{N^k}$$
  4. La série $\dsum P(T=j)$ converge (sa somme vaut $1$), plus précisement $$1=\dsum_{j\geq 1}P(T=j) =\dsum_{j\geq k}P(T=j)=\dsum_{n\geq 0} P(T=k+n).$$ On en déduit alors que la série $\dsum P(T>n)$ converge puisque $$\forall n\geq 1,\quad P(T>n) =\dfrac{N^k}{N-1} P(T=k+n)\leq P(T=k+n).$$ On en déduit que $T$ admet une espérance finie, et (voirs aussi exercice précédent), et $$E(T)=\dsum_{n\geq 0}P(T>n)=P(T>0)+\dsum_{n\geq 1}P(T>n) =1+\dsum_{n\geq 1}\dfrac{N^k}{N-1} P(T=k+n) =1+\dfrac{N^k}{N-1}\left(1-P(X=k)\right) $$ Soit $$\textcolor{blue}{\boxed{E(T)=\dfrac{N^k-1}{N-1}}}$$

($\star$) Correction

On lance une pièce équilibrée consécutivement. On s'arrête dès que deux piles successifs sont apparus. On note $X$ le nombre de lancers jusqu'à l'arrêt. Trouver la loi de $X$, s'agit-il d'une VA?

Correction

On note $P_n$ (resp. $F_n$) l'événement : "On a obtenu pile au $n$ème lancer" (resp. "On a obtenu face au $n$ème lancer").
Il est clair, que $$P(X=1)=0,\quad P(X=2)=P(P_1\cap P_2)=\dfrac{1}{4},\quad P(X=3)=P(F_1\cap P_2\cap P_3)=\dfrac{1}{8}.$$ Les événements ($F_1,\,P_1\cap P_2,\,P_1\cap F_2)$ forment un système complet, donc, pour tout $n\geq 3$, on a $$P(X=n)=P(F_1)P_{F_1}(X=n)+P(P_1\cap P_2)P_{P_1\cap P_2}(X=n)+P(P_1\cap F_2)P_{P_1\cap F_2}(X=n).$$ Or,

  1. $P(F_1)P_{F_1}(X=n)=\dfrac{1}{2}P(X=n-1)$, en effet la probabilité d'avoir $X=n$ sachant que la 1ère lancer est face est la même que $X=n-1$ (tous les lancers sont décalées de $1$).
  2. $P(P_1\cap P_2)P_{P_1\cap P_2}(X=n)=0$, puisque si on a $(P_1\cap P_2)$ est réalisé alors le jeux s'arrête.
  3. $P(P_1\cap F_2)P_{P_1\cap F_2}(X=n)=\dfrac{1}{4}P(X=n-2)$.

On en déduit, pour tout $n\geq 2$, $$P(X=n)=\dfrac{2P(X=n-1)+P(X=n-2)}{4}\Longrightarrow P(X=n)=\alpha \left(\frac{1+\sqrt{5}}{4}\right)^n+\beta \left(\frac{1-\sqrt{5}}{4}\right)^n.$$ On trouve, après calcul, $\alpha =\dfrac{\sqrt{5}-1}{2\sqrt{5}},\quad \beta\dfrac{\sqrt{5}+1}{2\sqrt{5}}$.

(CCP 2017) Correction

Une ligne de métro compte 4 arrêts. Le train met une minute pour aller d'une station à l'autre et s'y arrête pendant un temps négligeable. Un voyageur fatigué monte à la station $0$ au temps $t=0$ et s'endort; la VA $T$ associée au temps qu'il met à se réveiller suit une loi géométrique.
Trouver la loi suivi par $X$, VA représentant le numéro de la station où il se réveille.

Correction

L'événement $(X=0)$ peut s'écrire comme $\dsp \bigcup_{k\in \N^*} (T=4k)$, Donc $$P(X=0)=\dsum_{k\geq 1}p(1-p)^{4k-1}=\dfrac{p}{1-p}\dsum_{k\geq 1}(1-p)^{4k}=\dfrac{p}{1-p}\left(-1+\dfrac{1}{1-(1-p)^4}\right)=\dfrac{p(1-p)^3}{1-(1-p)^4}.$$ De même, $$\begin{array}{lcl} P(X=1)&=&\dsum_{k\in \N}p(1-p)^{(4k+1)-1}=\dfrac{p}{1-(1-p)^4}\\ P(X=2)&=&\dsum_{k\in \N}p(1-p)^{(4k+2)-1}=\dfrac{p(1-p)}{1-(1-p)^4}\\ P(X=3)&=&\dsum_{k\in \N}p(1-p)^{(4k+3)-1}=\dfrac{p(1-p)^2}{1-(1-p)^4}\\ \end{array}$$

(CCP 2017) Correction

On lance une pièce avec une probabilité $\frac23$ d'obtenir pile, les lancers étant indépendantes. On note $X$ la VA représentant le nombre de lancers nécessaire pour obtenir deux piles consécutifs.
Donner la loi de $X$ et calculer son espérance.

Correction

On note $P_n$ (resp. $F_n$) l'événement : "On a obtenu pile au $n$ème lancer" (resp. "On a obtenu face au $n$ème lancer").
Il est clair, que $$P(X=1)=0,\quad P(X=2)=P(P_1\cap P_2)=\dfrac{4}{9},\quad P(X=3)=P(F_1\cap P_2\cap P_3)=\dfrac{4}{27}.$$ Les événements ($F_1,\,P_1\cap P_2,\,P_1\cap F_2)$ forment un système complet, donc, pour tout $n\geq 3$, on a $$P(X=n)=P(F_1)P_{F_1}(X=n)+P(P_1\cap P_2)P_{P_1\cap P_2}(X=n)+P(P_1\cap F_2)P_{P_1\cap F_2}(X=n).$$ Or,

  1. $P(F_1)P_{F_1}(X=n)=\dfrac{1}{3}P(X=n-1)$, en effet la probabilité d'avoir $X=n$ sachant que la 1ère lancer est face est la même que $X=n-1$ (tous les lancers sont décalées de $1$).
  2. $P(P_1\cap P_2)P_{P_1\cap P_2}(X=n)=0$, puisque si on a $(P_1\cap P_2)$ est réalisé alors le jeux s'arrête.
  3. $P(P_1\cap F_2)P_{P_1\cap F_2}(X=n)=\dfrac{2}{9}P(X=n-2)$.

On en déduit, pour tout $n\geq 2$, $$P(X=n)=\dfrac{3P(X=n-1)+2P(X=n-2)}{9}\Longrightarrow P(X=n)=\alpha \left(\frac{2}{3}\right)^n+\beta \left(\frac{-1}{3}\right)^n.$$ On trouve, après calcul, $\alpha =\dfrac{2}{3},\quad \beta=\dfrac{4}{3}$.

($\star$) Correction

$n$ VA indépendantes $X_1,\cdots,X_n$ suivent chacune une loi de Bernoulli de paramètres respectives $1,\frac{1}{2},\cdots,\frac{1}{n}$.
Trouve la loi de la VA $N$ qui vaut $0$ si $X_1=X_2=\cdots=X_n=1$ et $\dsp \min\{k\in \inter{1,n},\,X_k=0\}$ sinon.

Correction

Il est clair que $P(N=0)=\dsp\prod_{i=1}^n P(X_i=1)=\dsp \prod_{i=1}^n\frac{1}{i}=\frac{1}{n!}.$ D'autre part, $P(N=1)=P(X_1=0)=0$, et $$\forall k\in \inter{2,n},\quad P(N=k)=\left(\prod_{i=1}^{k-1}P(X_i=1)\right)\,P(X_k=0)=\frac{1}{(k-1)!}\frac{k-1}{k}=\dfrac{1}{(k-2)!}\dfrac{1}{k}.$$

Independance

(Classique) Correction

Soit $X$ et $Y$ deux variables aléatoires indépendantes suivant des lois de Poisson de paramètres $\lambda $ et $\mu $. Reconnaître la loi de $X$ sachant $X + Y = n$.

Correction

Soit $k\in \N$ avec $k\leq n$, on a (en utilisant le faite que $X$ et $Y$ sont indépendantes), $$P_{(X+Y=n)}(X=k)=\dfrac{P(X=k \cap X+Y=n)}{P(X+Y=n)}=\dfrac{P(X=k)P(Y=n-k)}{P(X+Y=n)}.$$ Or $$P(X+Y=n)=\dsum_{k=0}^nP(X=k)P(Y=n-k)=\dsum_{k=0}^n\dfrac{\ee^{-\lambda}\lambda^k}{k!}\dfrac{\ee^{-\mu}\mu^{n-k}}{(n-k)!}=\ee^{-(\lambda+\mu)}\dfrac{(\lambda+\mu)^n}{n!} $$ On en déduit, $$\forall n\in \N,\quad P_{(X+Y=n)}(X=n)=\dfrac{\ee^{-\lambda}\lambda^k\ee^{-\mu}\mu^{n-k}}{k!(n-k)!}\dfrac{n!}{\ee^{-(\lambda+\mu)}(\lambda+\mu)^n}={n\choose k}\left(\dfrac{\lambda}{\lambda+\mu}\right)^k\left(\dfrac{\mu}{\lambda+\mu}\right)^{n-k}.$$ En notant $p=\dfrac{\lambda}{\lambda+\mu}$ donc $1-p=\dfrac{\mu}{\lambda+\mu}$, on reconnaît une loi binomiale de paramètres $n$ et $p$.

($\star$) Correction

Soit $X$ et $Y$ deux variables aléatoires discrètes indépendantes. On suppose que $X$ et $Y$ suivent des lois de Poisson de paramètres $\lambda $ et $\mu $. Quelle est la loi suivie par $X + Y$~?

Correction

Soit $n\in \N$, on a $(X+Y=n)=\dsp\bigcup_{k=0}^n (X=k\cup Y=n-k)$, donc $$\begin{array}{lcl} P(X+Y=n)&=&\dsum_{k=0}^nP(X=k)P(Y=n-k)=\dsum_{k=0}^n\dfrac{\ee^{-\lambda}\lambda^k}{k!}\dfrac{\ee^{-\mu}\mu^{n-k}}{(n-k)!}\\ &=&\dfrac{\ee^{-\lambda}\ee^{-\mu}}{n!}\dsum_{k=0}^n\dfrac{n!}{k!(n-k)!}\lambda^k\mu^{n-k}= \ee^{-(\lambda+\mu)}\dfrac{(\lambda+\mu)^n}{n!} \end{array}$$ On en déduit que $X+Y\hookrightarrow \mathcal{P}(\lambda+\mu)$.

($\star$) Correction

Soient $X$ et $Y$ deux variables aléatoires indépendantes géométriques de paramètres $p$ et $q$. Calculer l'espérance de $Z = \max (X,Y)$.

Correction

On suppose que $p,q\in ]0,1[$, soit $n\in \N^*$, on a $$(Z=n)=\left(X=n \cap Y < n\right) \cup \left( X< n \cap Y=n\right) \cup \left(X=n \cap Y=n\right).$$ En utilisant l'indépendance de $X$ et $Y$, on trouve: $$\begin{array}{lcl} P(Z=n)&=&P(X=n)\dsum_{k=1}^{n-1}P(Y=k)+P(Y=n)\dsum_{k=1}^{n-1}P(X=k)+P(X=n)P(Y=n)\\ &=&p(1-p)^{n-1}\left(1-(1-q)^{n-1}\right)+q(1-q)^{n-1}\left(1-(1-p)^{n-1}\right)+pq(1-p)^{n-1}(1-q)^{n-1}\\ &=&p(1-p)^{n-1}+q(1-q)^{n-1}-(p+q-pq)\left(1-(p+q-pq)\right)^{n-1} \end{array}$$ En notant $W$ la VAD t.q $W\hookrightarrow \mathcal{G}(p+q-pq)$, on trouve: $$E(Z)=E(X)+E(Y)-E(W)=\dfrac{1}{p}+\dfrac{1}{q}-\dfrac{1}{p+q-pq}.$$

($\star$) Correction

Soit $X$ et $Y$ deux variables aléatoires discrètes indépendantes. On suppose que celles-ci suivent des lois géométriques de paramètres $p$ et $q$.
Déterminer $P(X > n)$ pour $n \in \mathbb{N}$. En déduire la loi de $Z = \min (X,Y)$. La loi de $Z$ est-t-elle géométrique~?

Correction

Soit $n\in \N$, on a $(X>n)=\dsp{\bigcup_{k\geq n+1}}(X=k)$, ce qui donne $$P(X>n)=\dsum_{k=n+1}^\infty P(X=k)=\dsum_{k\geq n+1}p(1-p)^{k-1}=p\dsum_{k=0}^\infty (1-p)^{k+n}=(1-p)^n.$$ Soit $n\in \N^*$, on a $(Z=n)=\left( X>n\cap Y=n\right)\cup \left(X=n \cap Y>n\right)\cup \left( X=n \cap Y=n\right)$, vu que $X$ et $Y$ sont indépendantes, on conclut: $$\begin{array}{lcl} P(Z=n)&=&P(X>n)P(Y=n)+P(X=n)P(Y>n)+P(X=n)P(Y=n)\\ &=&(1-p)^nq(1-q)^{n-1}+p(1-p)^{n-1}(1-q)^n+pq(1-p)^{n-1}(1-q)^{n-1}\\ &=&\left((1-p)(1-q)\right)^{n-1}\left(q(1-p)+p(1-q)+pq\right)\\ &=& (p+q-pq)\left((1-p)(1-q)\right)^{n-1}=(p+q-pq)\left(1-(p+q-pq)\right)^{n-1}. \end{array}$$ On en déduit que $Z$ est une VAD qui suit une loi géométrique de paramètre $p+q-pq$.

($\star\star$) Correction

Soit $(X_n)$ une suite de variables aléatoires mutuellement indépendantes suivant la même loi de Bernoulli de paramètre $p \in ]0, 1[$. On note pour tout $n$ dans $\N^*$ : $$S_n =\dfrac{X_1+\cdots+X_n}{n},\quad Y_n =\dfrac{X_n+X_{n+1}}{2},\quad T_n=\dfrac{Y_1+\cdots+Y_n}{n}.$$

  1. Montrer que, pour tout $\varepsilon>0$, on a $P\left(\abs{S_n-p}\right)\tendversN\,0$.
  2. Déterminer pour tout $n \in \N^*$ la loi et l'espérance de $Y_n$ ; pour $m < n$, $Y_m$ et $Y_n$ sont-elles indépendantes?
  3. Montrer que, pour tout $\varepsilon>0$, $P\left(\abs{T_n-p}\right)\tendversN\,0$.

Correction

  1. Application direct du théorème du cours.
  2. Il est clair que si $\abs{n-m}\geq 2$ alors $Y_n,\,Y_m$ sont indépendants.
    Par contre si $m=n+1$, alors $Y_n,\,Y_{n+1}$ ne sont pas indépendantes, en effet,
    1. $P(Y_{n+1}=0)=P(X_{n+1}=0\cap X_{n+2}=0)=(1-p)^2$, et $P(Y_n=1)=P(X_n=1\cap X_{n+1}=1)=p^2$, donc $$P(Y_{n+1}=0)P(Y_n=1)=p^2(1-p)^2$$
    2. D'autre part, $$P((Y_{n+1}=0)\cap (Y_n=1))=P(((X_{n+2}=0) \cap (X_{n+1}=0))\cap ((X_{n+1}=1) \cap (X_{n}=1))$$ donc $P((Y_{n+1}=0)\cap (Y_n=1))=0$ puisque $(X_{n+1}=1) \cap (X_{n+1}=0)= \emptyset$.
    Ceci montrer $P((Y_{n+1}=0)\cap (Y_n=1))\neq P(Y_{n+1}=0)P(Y_{n}=1)$.
  3. Voir exercice précédent.

($\star\star\star$ + Central ) Correction

Soit $(X_n)$ une suite de variables aléatoires indépendantes à valeur dans $\N$, de même loi $X$, d'espérance $m$ et de variance $\sigma^2$. Soit $N$ une variable aléatoire à valeur dans $\N^*$ admettant une espérance et une variance finies. On pose $S=\dsum_{k=0}^N X_k$,

  1. Montrer que $S$ admet une espérance finie, et que $E(S)=E(N)E(X)$.
  2. Montrer que $S$ admet une variance et que $V(S)=V(X)E(N)+E(X)^2V(N)$.

Correction

Pour $n\in \N^*$, on note $S_n=\dsum_{k=0}^nX_k$, pour évaluer la probabilisé de l'événement $(S=k)$ (avec $k\in \N$), on peut alors écrire: $(S=k)= \dsp\bigcup_{n\in \N^*} ((N=n) \cap (S_n=k))$. Ce qui donne $$\forall k\in \N,\quad P(S=k)=\dsum_{n\geq 0} P((N=n) \cap (S_n=k))=\dsum_{n\geq 1} P(N=n)P(S_n=k).$$ Montrons que la famille $(kP(N=n)P(S_n=k))_{(n,k)\in \N^*\times\N}$ est sommable.
Soit $n\in \N^*$, la série $\dsum k P(N=n)P(S_n=k)$ est convergente, en effet, puisque $X_j$ admet une espérance finie $E(X)$ donc $S_n$ admet une espérance finie (et $E(S_n)=nE(X)$) ce qui implique que $$ \dsum_{k\geq 0} k P(N=n)P(S_n=k)\text{ converge, et } \dsum_{k\geq 0} k P(N=n)P(S_n=k)=P(N=n) E(S_n)=nP(N=n)E(X).$$ D'autre part la série $\dsum nP(N=n)$ converge car $N$ admet une espérance fini, et $\dsum nP(N=n)=E(N)$.
Ainsi la famille $(kP(N=n)P(S_n=k))_{(n,k)\in \N^*\times\N}$ est sommable. Donc $$\dsum_{(n,k)\in \N^*\times\N}(kP(N=n)P(S_n=k))=\dsum_{n\geq 1}\dsum_{k\geq 0}kP(N=n)P(S_n=k)=E(X)\dsum_{n\geq 1}nP(N=n)=E(X)E(N).$$ D'autre par, ceci prouve que la série $\dsum_k kP(S=k)$ converge, donc $S$ admet une espérance finie, et on a $$E(S)=\dsum_{k\geq 0}kP(S=k)= \dsum_{(n,k)\in \N^*\times\N}(kP(N=n)P(S_n=k))=E(X)E(N).$$

(Mines 2017) Correction

Soient $(X_i)_{1\leq i\leq n}$, $n\,$ VAD mutuellement indépendantes. Pour $i\in \inter{1,n}$, on pose $Y_i=\dsum_{k=1}^iX_i$.
Expliciter $M=\left(\mathrm{Cov}(Y_i,Y_j)\right)_{i,j\in \inter{1,n}}$ et l'exprimer en fonction de $A=\begin{pmatrix} 1&\cdots&1\\ &\ddots&\\ 0&&1 \end{pmatrix}$. Proposer un encadrement de valeurs propres de $M$.

Correction

Comme $\mathrm{Cov}(X,Y)=\mathrm{Cov}(Y,X)$ on en déduit que la matrice $M$ est symétrique.
Soit $(i,j)\in \inter{1,n}^2$, on suppose que $j\geq i$, en utilisant la bilinéarité de la covariance, on trouve: $$\mathrm{Cov}(Y_i,Y_j)= \mathrm{Cov}(\dsum_{k=1}^iX_k,\dsum_{s=1}^jX_s)=\dsum_{k=1}^i\dsum_{s=1}^j\mathrm{Cov}(X_k,X_s)=\dsum_{k=1}^i\dsum_{s=1}^j \delta_{k,\,s}\mathrm{Cov}(X_k,X_s)$$ Autrement dit, avec l'hypothèse $j\geq i$, $\mathrm{Cov}(Y_i,Y_j)=i\mathrm{Cov}(X,X)=iK$, donc $M$ est de la forme $$M=K\begin{pmatrix} 1&1&1&1&\cdots&1&1\\ 1&2&2&2& &&2\\ 1&2&3&3& &&3\\ 1 &2 &3 &4& & &4\\ & & & & \ddots & &\vdots\\ \vdots & & & \vdots & & n-1&n-1\\ 1&2&3&4 & \cdots &n-1&n \end{pmatrix}=K\, ( {\,}^t A \, A)$$ La matrice $M$ étant symétrique, donc diagonalisable. Notons $\Sp (A)=\{\lambda_1,\cdots,\lambda_n\}$. Soit $k\in \inter{1,n}$ et $Z_k={\,}^t (\alpha_1,\cdots,\alpha_n)\in \R^n$ un vecteur propre associé à $\lambda_k$ (donc $Z_k\neq 0$, et quitte à diviser $Z_k$ par une constante, on peut supposer que $\norme{Z_k}_2=1)$. On a alors, $$MZ_k=\lambda_kZ_k\Longrightarrow K{\,}^t A AZ_k =\lambda_kZ_k\Longrightarrow {\,}^t Z_k{\,}^t A_kAZ_k=\dfrac{\lambda_k}{K}{\,}^t Z_kZ_k$$ Autrement dit, $\norme{AZ_k}^2=\dfrac{\lambda_k}{K}$ ce qui montre que $\lambda_k>0$, d'autre part, $$AZ_k=\begin{pmatrix} \alpha_1+\alpha_2+\cdots+\alpha_n\\ \alpha_2+\cdots+\alpha_n\\ \vdots\\ \alpha_{n-1}+\alpha_n\\ \alpha_n \end{pmatrix}\Longrightarrow \norme{AZ_k}^2\leq \max_{i\in \inter{1,n}}(\abs{\alpha_i})\dsum_{j=1}^nj^2\leq \dfrac{n(n+1)(2n+1)}{6}$$ Donc $\lambda_k\leq \dfrac{K}{6}(2n^3+3n^2+n)$.

(CCP 2017) Correction

Soit $(X_n)$ une suite de variables aléatoires mutuellement indépendantes suivant la même loi de Bernoulli de paramètre $p \in ]0, 1[$. On note pour tout $n$ dans $\N^*$ : $ Y_n =\dfrac{X_n+X_{n+1}}{2},\quad S_n=\dsum_{i=1}^nY_i.$

  1. Calculer $E(S_n),\,V(S_n)$.
  2. Montrer que, pour tout $\varepsilon>0$, $P\left(\abs{\frac{S_n}{n}-p}\geq \varepsilon\right)\tendversN\,0$.

Correction

  1. Soit $n\geq 1$, on a $$\begin{array}{lcl} S_n&=&Y_1+Y_2+\cdots+Y_{n-1}+Y_n\\ &=&\dfrac{1}{2}\left(X_1+X_2+X_2+X_3+\cdots+ X_{n-1}+X_n+X_n+X_{n+1}\right)\\ &=&\dfrac{1}{2}\left(X_1+X_{n+1}\right)+\dsum_{k=2}^nX_k. \end{array}$$ Par linéarité de l'espérance, on obtient, $$E(S_n)=\dfrac{1}{2}\left(E(X_1)+E(X_{n+1})\right)+\dsum_{k=2}^nE(X_k)=p+(n-1)p=np.$$ De même, on a $$V(S_n)=\dfrac{1}{4}\left(V(X_1)+V(X_{n+1}\right)+\dsum_{k=2}^nV(X_k)=\dfrac{4n-2}{4}(p(1-p)).$$
  2. On applique le théorème de Bienaymé-Tchebychev à $Z_n=\dfrac{S_n}{n}$, ce qui donne $$0\leq P\left(\abs{Z_n-E(Z_n)}\geq \varepsilon\right)= P\left(\abs{\frac{S_n}{n}-p}\geq \varepsilon\right)\leq \dfrac{V(Z_n)}{\varepsilon^2}=\dfrac{(4n-2)p(1-p)}{4n^2\varepsilon^2}$$

(TPE-EIVP PSI 2018) Correction

Soit $(a,b,c)\in\mathbb{R}^3$ et $M=\left( \begin{array}{ccc}a&b&c\\ a&b&c\\ a&b&c \end{array} \right)$.

  1. Donner une condition nécessaire et suffisante pour que $M$ soit diagonalisable.
  2. Donner une condition nécessaire et suffisante pour que $M$ soit une matrice de projecteur.
  3. Soient $X, Y$ et $Z$ trois variables aléatoires indépendantes, suivant la même loi binomiale $\mathcal{B}(n,p)$. Pour tout $\omega\in\Omega$, soit $M(\omega)=\left( \begin{array}{ccc}X(\omega)&Y(\omega)&Z(\omega)\\ X(\omega)&Y(\omega)&Z(\omega)\\X(\omega)&Y(\omega)&Z(\omega) \end{array} \right)$.
  4. Quelle est la probabilité que $M$ soit diagonalisable ?
  5. Quelle est la probabilité que $M$ soit une matrice de projecteur ?

Correction

  1. Notons $U=\begin{pmatrix} 1\\1\\1 \end{pmatrix}$ et $V=\begin{pmatrix} a\\ b\\c \end{pmatrix}$ alors $M= U\,{\,}^t V$.
    Si $V\neq 0$ alors soit $(V_1,V_2)$ une base de $(V)^\perp$, on a $$\forall i\in \{1,2\},\quad MV_i=U\,{\,}^t V\,V_i=\scal{V}{V_i}U=0$$ donc $0$ une valeur propre de $M$ d'ordre au moins 2. Vu que la somme des valeurs propres égale la trace, alors la deuxième valeur propre est $a+b+c$.
    Donc $M$ est diagonalisable ssi $a+b+c\neq 0$.
    Si $V=0$ alors $M=0$ donc diagonalisable.
  2. $M$ la matrice d'un projecteur ssi $M^2=M$, or $$M^2=(U\,{\,}^t V)(U\,{\,}^t V)=U ({\,}^tV U) {\,}^t V=\scal{V}{U} U\,{\,}^t V=(a+b+c) M$$ ainsi $M$ est la matrice d'un projecteur ssi ($a+b+c=\tr(M)=1$ ou $a=b=c=0$).
  3. $M$ n'est pas diagonalisable ssi $(X,Y,Z)\neq (0,0,0)$ et $X+Y+Z=0$, or vu que $X\geq 0,\,Y\geq 0,\,Z\geq 0$ alors $X+Y+Z=0$ ssi $X=Y=Z=0$, et dans ce cas $M$ est diagonalisable.
    Conclusion: $P(M \text{ soit diagonalisable})=1$.
  4. $M(w)$ est la matrice d'un projecteur ssi $X+Y+Z=1$ ou $X=Y=Z=0$. Donc la probabilité de $M^2=M$ est égale à la probabilité de l'événement $(X=Y=Z=0) \cup (X+Y+Z=1)$, vu que $$(X+Y+Z=1)= (X=1 \cap Y=0\cap Z=0 )\cup (X=0 \cap Y=1\cap Z=0 )\cup (X=0 \cap Y=0\cap Z=1 )$$ on en déduit,

($\star$) Correction

Soit $(X_n)_{n\geq 1}$ une suite de variables aléatoires deux à deux indépendantes. On suppose que chaque $X_n$ suit une loi de Bernoulli de paramètre $p_n$. On note $S_n=X_1+\dots+X_n$ et on souhaite démontrer que, $$ \forall \varepsilon>0, \lim_{n\to+\infty}P\left(\left|\frac {S_n}n-\frac 1n\sum_{k=1}^n p_k\right|\geq \varepsilon\right)=0.$$

  1. Pourquoi ne peut-on pas appliquer directement la loi faible des grands nombres?
  2. Quelle est l'espérance de $S_n$? Sa variance? Démontrer que $V(S_n)\leq n$.
  3. En déduire le résultat.

Correction

  1. Pour pouvoir appliquer la loi faible des grands nombres, il faut que les v.a.d $X_n$ ont la même espérance, or ce n'est pas le cas ici (sauf si on suppose que $p_1=\cdots=p_n$).
  2. Comme $X_k\hookrightarrow \Bb(p_k)$, alors $E(X_k)=p_k$ et $V(X_k)=p_k(1-p_k)$, on en déduit alors, $$E(S_n)=\dsum_{k=1}^nE(X_k)=\dsum_{k=1}^pp_k,\cdots V(S_n)=\dsum_{k=1}^nV(X_k)=\dsum_{k=1}^np_k(1-p_k)\leq n.$$
  3. On note $Y=\dfrac{S_n}{n}$, alors $Y$ admet une espérance et une variance, et on a $$E(Y)=\dfrac{1}{n}E(S_n)=\dfrac{1}{n}\dsum_{k=1}^np_k,\quad V(Y)=\dfrac{1}{n^2}V(S_n)\leq \dfrac{1}{n}.$$ On applique ensuite l'inégalité de Bienaymé-Tchebychev (Thm \ref{InegBT}). Ce qui donne, $$\forall \varepsilon>0,\,P(\abs{Y-E(Y)}\geq \varepsilon)=P\left(\abs{\dfrac{S_n}{n}-\dfrac{1}{n}\dsum_{k=1}^np_k}\geq \varepsilon\right)\leq \dfrac{V(Y)}{\varepsilon^2}\leq \dfrac{1}{n\varepsilon^2}.$$ $\varepsilon$ étant fixé, donc $\dfrac{1}{n\varepsilon^2}\tendversN\,0$, d'autre part, $P\geq 0$, donc en utilisant le théorème d'encadrement on obtient le résultat.

($\star$) Correction

Soit $X$ une VAD tel que $X(\Omega)=\{-1,0,1\}$ et $P(X=-1)=P(X=0)=P(X=1)=1/3$. On pose $Y=X^2$.

  1. Calculer $E(X),\,E(Y),\,E(XY)$.
  2. $X,\,Y$ sont indépendantes?

Correction

  1. On a $E(X)= -1\times\dfrac{1}{3}+0\times\dfrac{1}{3}+\dfrac{1}{3}=0$.
    $Y(\Omega)=\{0,1\}$ et $(Y=0)=(X=0),\, (Y=1)=(X=-1)\cup (X=1)$. On en déduit, $$P(Y=0)=P(X=0)=\dfrac{1}{3},\,P(Y=1)= 1-P(Y=0)=\dfrac{2}{3}\Longrightarrow E(Y)=\dfrac{2}{3}$$ Pour calculer $E(XY)$ il suffit de remarque que $XY=X^3 =X$ puisque $X$ ne prend que des valeurs $\alpha$ tel que $\alpha=\alpha^3$. On en déduit, $$E(XY)=0=E(X)E(Y) \Longrightarrow \mathrm{Cov}(X,Y)=0.$$
  2. $P((Y=1)\cap (X=1))=P(X=1)=\dfrac{1}{3}$ tandis que $P(X=1)P(Y=1)=\dfrac{1}{3}\dfrac{2}{3}=\dfrac{2}{9}$. On en déduit que $X$ et $Y$ ne sont pas indépendantes.

(Agrég 2019 mais niveau PSI) Correction

$X$ et $Y$ désignent deux variables aléatoires discrètes à valeurs dans $\N$ et indépendantes.

  1. Montrer que $P(X=Y)=\dsum_{k=0}^\infty P(X=k)P(Y=k)$.

On suppose à partir de maintenant que $X\hookrightarrow \mathcal{P}(\lambda)$ avec $\lambda>0$ et qu'il existe $p\in ]0; 1[$ tel que, pour tout $k\in \N, P (Y = k) = p(1 - p)^k$. On considère la matrice aléatoire $A=\begin{pmatrix} X&X+Y\\0&Y \end{pmatrix}$.

  1. Calculer la probabilité que $A$ ne soit pas inversible.
  2. Préciser la loi de la variable aléatoire $\rg (A)$ (qui donne le rang de la matrice $A$) ainsi que son espérance.
  3. Donner une condition nécessaire et suffisante sur $a; b$ et $c$ pour qu'une matrice de $\MM_2(\R)$; de la forme $\begin{pmatrix} a& c\\ 0& b \end{pmatrix}$ soit diagonalisable.
  4. Calculer la probabilité que $A$ soit diagonalisable.

Correction

  1. L'événement $(X=Y)$ s'écrit sous la forme $\dsp\bigcup_{k\in \N}(X=k)\cap (Y=k)$ et vu que ces événement sont incompatibles et que $X,\,Y$ sont indépendants, on en déduit, $$P(X=Y)=\dsum_{k=0}^\infty P((X=k)\cap (Y=k))=\dsum_{k=0}^\infty P(X=k)P(Y=k).$$
  2. $A$ n'est pas inversible ssi $X=0$ ou $Y=0$, donc $$\begin{array}{lcl} P(A \text{ n'est pas inversible})&=& P((X=0)\cup (Y=0))\\ &=&P(X=0)+P(Y=0)-P((X=0)\cap (Y=0))\\ &=&P(X=0)+P(Y=0)-P(X=0)P(Y=0)\\ &=&\ee^{-\lambda}+ p-p\ee^{-\lambda}. \end{array}$$
  3. Comme $A$ est de taille 2, alors les possibilité de rang de $A$ sont 0,1 et 2.
    1. $\rg(A)=2$ si $A$ est inversible, donc $$P(\rg (A)=2)=1-P(A \text{ n'est pas inversible}) =1-(\ee^{-\lambda}+ p-p\ee^{-\lambda}).$$
    2. $\rg(A)=0$ ssi $A=0$ donc $X=Y=0$ $$P(\rg (A)=0)=P((X=0)\cap (Y=0))=P(X=0)P(Y=0)=p\ee^{-\lambda}.$$
    3. Enfin $\rg (A) =1$ ssi $\rg(A)\not\in \{0,2\}$, donc $$P(\rg (A)=1)=1-P(\rg(A)\in \{0,2\})=1-p\ee^{-\lambda}-(1-(\ee^{-\lambda}+ p-p\ee^{-\lambda}))= \ee^{-\lambda}+p-2p\ee^{-\lambda}.$$
  4. D'après le cours d'algèbre, on a $A$ est diagonalisable ssi ($a\neq b$) ou ($a=b$ et $c=0$).

(Mines 2015) Correction

Soient $X,\,Y$ deux VAD indépendantes suivant une loi géométrique de même paramètre $p\in ]0,1[$. Donner la loi de $\abs{X-Y}$.

Correction

On pose $Z=\abs{X-Y}$, alors $Z(\Omega)=\N$. $$P(Z=0)=P(X=Y)=\dsum_{k=1}^\infty P(X=k,\,Y=k)=\dsum_{k=1}^\infty p^2(1-p)^{2(k-1)}=\dfrac{p^2}{1-(1-p)^2}.$$ Soit $k\in \N^*$, l'événement $(Z=k)$ peut s'écrire sous la forme $$(Z=k)= (X-Y=k) \cup (Y-X=k)= \bigcup_{n\in \N^*} (X=n+k,\,Y=n)\cup (Y=n+k,\,X=n).$$ Comme $X,\,Y$ sont indépendantes et les événements ci-dessous sont incompatibles, alors: $$ \begin{array}{lcl} P(Z=k)&=&\dsum_{n\in\N} P(X=n+k)P(Y=n)+P(X=n)P(Y=n+k)\\ &&\\ &=& \dsum_{n\in\N}p(1-p)^{n+k-1}p(1-p)^{n-1}+p(1-p)^{n-1}p(1-p)^{n+k-1}=2p^2(1-p)^{k}\dsum_{n\geq 1} (1-p)^{2n-2}\\ &=&\dfrac{2p^2(1-p)^{2k}}{1-(1-p)^2}=\dfrac{2p(1-p)^k}{2+p} \end{array} $$

($\star\star$) Correction

Soit $n\in \N^*$. On considère une suite de VAD $(X_k)_{k\in \N^*}$ indépendantes de même loi : $X_k\hookrightarrow \mathcal{U}(\inter{1,n})$. On souhaite trouver le nombre moyen de tirages nécessaire pour obtenir tous les numéros de $1$ à $n$.

On note $T_k$ le nombre de tirages nécessaire pour obtenir $k$ numéro différents $(k\in \inter{1,n})$ et $Z_1=T_1$, $Z_k=T_k-T_{k-1}$ pour $k\in \inter{2,n}$.

  1. Montrer que, pour $k\in \inter{2,n}$, $Z_k\hookrightarrow \mathcal{G}(p_k)$.
  2. En déduire un équivalent de $\mathrm{E}(T_n)$.
Correction

  1. Soit $k\in \inter{2,n}$, soit $j\in \N^*$, on écrit $$P(Z_k=j)=\dsum_{i=1}^\infty P(Z_k=j\cap T_{k-1}=i)=\dsum_{i\geq 1}P(T_k=j+i\cap T_{k-1}=i)=\dsum_{i\geq 1}P(T_k=j+i \mid T_{k-1}=i)P(T_{k-1}=i).$$ Or si $T_{k-1}=i$, $T_k=i+j$ signifie que les $j-1$ tirages $i+1,\cdots,i+j-1$ ont donné des numéros parmi les $k-1$ numéros distincts déjà tirés et le tirage $i+j$ a donné un numéro parmi les $n-k+1$ numéros restants. Donc: $$P(T_k=j+i \mid T_{k-1}=i) =\left(\dfrac{k-1}{n}\right)^{j-1}\left(\dfrac{n-k+1}{n}\right).$$ Soit : $$\begin{array}{lcl} P(Z_k=j)&=&\dsum_{i=1}^\infty \left(\dfrac{k-1}{n}\right)^{j-1}\left(\dfrac{n-k+1}{n}\right)P(T_{k-1}=i)\\ &&\\ &=& \left(\dfrac{k-1}{n}\right)^{j-1}\left(\dfrac{n-k+1}{n}\right)\dsum_{i=1}^\infty P(T_{k-1}=i)=\left(\dfrac{k-1}{n}\right)^{j-1}\left(\dfrac{n-k+1}{n}\right). \end{array}$$ Ainsi, $Z_k\hookrightarrow \mathcal{G}(p_k)$ avec $p_k=\left(1-\dfrac{k-1}{n}\right)$.
  2. On a $$T_n=T_n-T_{n-1}+T_{n-1}-\cdots +T_2-T_1+T_1=\dsum_{k=1}^n Z_k$$ Puisque les $Z_k$ ont une espérance finie, alors il en va de même pour $T_n$, de plus, $$\mathrm{E}(T_n)=\mathrm{E}\left(\dsum_{k=1}^n Z_k\right)=\dsum_{k=1}^n Z_k\mathrm{E}(Z_k)=\mathrm{E}(Z_1)+\dsum_{k=2}^n\dfrac{n}{n-k+1}.$$ Or $Z_1=T_1$, et $T_1$ est la VAD constante (égale à $1$), donc $\mathrm{E}(Z_1)=1$, soit $$\mathrm{E}(T_n)=1+\dsum_{k=2}^n\dfrac{n}{n-k+1}=1+n\dsum_{k=2}^n\dfrac{1}{k}=n\dsum_{k=1}^n\dfrac{1}{k}$$ Ainsi, $\mathrm{E}(T_n) \underset{n\to \infty}{ \thicksim}n\ln (n)$, on peut aussi affiner l'équivalence, en effet, on a : $$\dsum_{k=1}^n\dfrac{1}{k}=\ln(n)+\gamma +\underset{n\to \infty}{\mathrm{o}}(1)\Longrightarrow \mathrm{E}(T_n) \underset{n\to \infty}{ \thicksim}n\ln (n)+\gamma n.$$ Avec $\gamma\simeq \,0.57721$.

Loi

($\star\star$) Correction

Soit $X$ une variable aléatoire à valeurs dans $\N$ telle que $p_n=P(X=n)>0$. On note alors $z_n=P_{(X\geq n)}(X=n)$. Montrer que la loi de $X$ est définie par la suite $(z_n)$. Quelle est la loi de $X$ si la suite $(z_n)$ est constante?
On suppose que $p_n={n+2 \choose 2} p^3(1-p)^n,\, \,p\in ]0,1[$. Montrer que $X$ est une variable aléatoire, trouver la suite $(z_n)$ dans ce cas et calculer $\limiteX{n}{\infty} z_n$.

Correction

Soit $n\in \N$, on a $$z_n=P_{(X\geq n)}(X=n)=\dfrac{P(X\geq n \cap X=n)}{P(X\geq n)}=\dfrac{P(X=n)}{\dsum_{k=n}^\infty P(X=k)}=\dfrac{p_n}{\dsum_{k=n}^\infty p_k}=\dfrac{p_n}{1-(p_0+\cdots+p_{n-1})}.$$ Montrons que la loi de $X$ peut être déterminée par la suite $(z_n)$. $$z_0=\dfrac{p_0}{1}\Longrightarrow p_0=z_0,\,z_1=\dfrac{p_1}{1-p_0}\Longrightarrow p_1=z_1(1-z_0)$$ de même, $z_2=\dfrac{p_2}{1-p_0-p_1}$ ce qui donne $p_2=z_2(1-z_0-z_1(1-z_0))=z_2(1-z_0)(1-z_1)$, on montre alors par récurrence sur $n\in \N$, $$p_0=z_0,\quad \forall n\geq 1,\,p_n=z_n\prod_{k=0}^{n-1}(1-z_k).$$ Dans le cas ou la suite $(z_n)$ est constante, alors, pour tout $n\geq 1$, on a $p_n=p_0(1-p_0)^{n}$. Donc $X+1\hookrightarrow\mathcal{G}(p_0)$.
Avant de traiter l'exemple donné, rappelons le résultat suivant, $$\forall x\in ]-1,1[,\quad \dsum_{k\geq 0}x^k=\dfrac{1}{1-x},\quad \dsum_{k\geq 0}(k+1)x^k=\dfrac{1}{(1-x)^2},\quad \dsum_{k\geq 0}(k+2)(k+1)x^k=\dfrac{2}{(1-x)^3}.$$ Comme $p\in ]0,1[$, alors $p_n\geq 0$ pour tout $n\in \N$ et $\dsum p_n$ converge, de plus, $$\dsum_{n\geq 0} p_n=\dsum_{n\geq 0}{n+2 \choose 2} p^3(1-p)^n=\dfrac{p^3}{2}\dsum_{n\geq 0}(n+2)(n+1)p^n=\dfrac{p^3}{2}\dfrac{2}{\left(1-(1-p)\right)^3}=1.$$ Donc $X$ est une VAD. Soit $n\geq 0$, calculons $P(X\geq n)$, $$\begin{array}{lcl} P(X\geq n)&=&\dfrac{p^3}{2}\dsum_{k=n}^\infty (k+2)(k+1)(1-p)^k=\dfrac{p^3(1-p)^n}{2}\dsum_{k=0}^\infty (k+2+n)(k+1+n)(1-p)^k\\ &=&\dfrac{p^3(1-p)^n}{2}\dsum_{k=0}^\infty \left((k+2)(k+1)+2n(k+1)+n^2+n\right)(1-p)^3\\ &=&\dfrac{p^3(1-p)^n}{2}\left(\dfrac{2}{p^3}+\dfrac{2n}{p^2}+\dfrac{n^2+n}{p}\right)= \dfrac{(1-p)^n}{2}\left(2+2np+(n^2+n)p^2\right) \end{array}$$ On en déduit, $$\forall n\in \N,\quad z_n=\dfrac{ p^3(n+2)(n+1)(1-p)^n}{2}\dfrac{2}{(1-p)^n\left(2+2np+(n^2+n)p^2\right)}=\dfrac{p^3(n+2)(n+1)}{\left(2+2np+(n^2+n)p^2\right)}\tendversN\,p.$$

($\star$) Correction

Soit $(x_n)_n$ une suite réelle telle que, pour tout $n\in \N$, on a $4x_{n+2}-5x_{n+1}+x_n=0$. Peut-t-on définir une variable aléatoire, à valeur dans $\N$, telle que, pour tout $n$, $P(X=n)=x_n$?.

Correction

Pour que cette suite définisse une variable aléatoire, il faut: $$\mathbf{a)\,} \quad \forall n\in \N,\,x_n\geq 0,\quad \mathbf{b)\,}\quad \dsum_{n\geq 0} x_n=1.$$ Les solutions de l'équation caractéristique $4x^2-5x+1$ sont $1$ et $\frac{1}{4}$, on en déduit qu'il existe $\alpha,\beta\in \R$ tels que, pour tout $n\in \N$, on ait $x_n=\alpha+\beta \left(\frac{1}{4}\right)^n$.
La condition $\mathbf{b)\,}$ implique que $\limiteX{n}{\infty}x_n=0=\alpha$, donc $x_n=\beta \left(\frac{1}{4}\right)^n$, puis $\dsum_{n\geq 0}x_n=1$ implique $\beta =\frac{3}{4}$. Si on prend $\alpha=0$ et $\beta =\frac{3}{4}$ alors les conditions $\mathbf{a)\,},\,\mathbf{b)\,}$ sont vérifiées.

($\star$) Correction

Soient $X$ et $Y$ deux variables aléatoires indépendantes et à valeurs dans $\N$. Elles suivent la même loi définie par : $\forall k\in\N,\,\, P(X = k) = P(Y = k) = pq^k$ où $p \in ]0, 1[$ et $q = 1- p$. On considère alors les variables $U$ et $V$ définies par $U = \sup(X, Y )$ et $V = \inf(X, Y )$.

  1. Déterminer la loi du couple $(U; V )$.
  2. Expliciter les lois marginales de $U$ et de $V$ .
  3. $U$ et $V$ sont-elles indépendantes ?

Correction

  1. Soient $(n,m)\in \N^2$, alors,
    1. Si $m>n$ alors $P\left( (U=n)\cap (V=m)\right)=0$ puisque $U\geq V$.
    2. Si $n=m$, alors $P\left( (U=n)\cap (V=n)\right)=P\left( (X=n)\cap (V=n)\right)$, comme $X$ et $Y$ sont indépendantes alors, $P\left( (U=n)\cap (V=n)\right)=P\left(X=n\right)P\left(Y=n\right)=p^2q^{2n}$.
    3. Enfin si $m< n)$, alors $$P\left( (U=n)\cap (V=m)\right)=P\left( \left((X=n)\cap (Y=m)\right)\cup \left( (X=m)\cap (Y=n)\right)\right)=p^2q^{n+m}+p^2q^{m+n}=2p^2q^{n+m}.$$
  2. Soit $k\in \N$, on a $P(U=k)=\dsum_{m\in \N}P\left(U=k \cap V=j\right)$, en utilisant la question précédente, on trouve: $$P(U=k)=\dsum_{m=0}^kP\left(U=k \cap V=j\right)=\dsum_{m=0}^{k-1}2p^2q^{k+j}+p^2q^{2k}=2p^2q^k\dfrac{1-q^k}{1-q}+p^2q^{2k}=2pq^k(1-q^k)+p^2q^{2k}.$$ De même, $$\begin{array}{lcl} P(V=k)&=&\dsum_{j=0}^\infty P\left(U=j \cap V=k\right)\\ &=&P\left(U=k \cap V=k\right)+\dsum_{j=k+1}^\infty P\left(U=j \cap V=k\right)=p^2q^{2k}+\dsum_{j=k+1}^\infty 2p^2q^{k+j}\\ &=&p^2q^{2k}+2p^2q^{2k+1}\dfrac{1}{1-q}=p^2q^{2k}+2pq^{2k+1} \end{array}$$

($\star$) Correction

Soient $X$ et $Y$ deux variables aléatoires définies sur un même espace probabilisé $(\Omega, \mathcal{F}, P)$ et à valeurs dans $\N$. On suppose que la loi du couple $(X; Y )$ est donnée par : $$\forall (i; j) \in \N^2,~~ P((X = i) \cap (Y = j)) = \dfrac{1}{\ee 2^{i+1}j!}.$$

  1. Déterminer les lois de $X$ et de $Y$ .
  2. Prouver que $1 + X$ suit une loi géométrique et en déduire l'espérance et la variance de $X$. Déterminer l'espérance et la variance de $Y$.
  3. Les variables $X$ et $Y$ sont-elles indépendantes ? Calculer $P(X = Y )$.

Correction

  1. $$\forall k\in \N,~~P(X=k)=\dfrac{1}{2^{k+1}},\quad P(Y=k)=\dfrac{1}{k!\,\ee}.$$
  2. On pose $Z=X+1$, alors $Z(\Omega)=\N^*$, $P(Z=n)=\dfrac{1}{2}\dfrac{1}{2^{n-1}}$.
    $E(Z)=2,\,V(Z)=2$ donc $E(X)=1,\,V(X)=2$.
  3. $P(X=Y)=\dfrac{1}{2\sqrt{\ee}}$

($\star$) Correction

Soit $X$ une variable aléatoire suivant la loi de Poisson de paramètre~$\lambda$

  1. Donner la loi de $X^2+1$.
  2. Calculer $P(2X < X^2+1)$.
  3. Calculer $P(X$ est pair$)$.
  4. Soit $Y$ une variable aléatoire sur le même espace probabilisé, indépendante de~$X$, prenant les valeurs $1$ et $2$ avec probabilité~$\frac12$.
    Calculer $P(XY$ est pair$)$.

Correction

  1. Soit $n\in \N$, s'il existe $k\in \N$ tel que $n=k^2+1$ alors $P(X^2+1=n)=P(X=k)=\dfrac{\ee^{-\lambda}\lambda^k}{k!}$, sinon $P(X^2+1=n)=0$.
  2. $P(2X < X^2+1)=P((X-1)^2>0)=P(X\neq 1)=1-P(X=1)=1-\lambda\ee^{\lambda}$.
  3. $P(X\text{ est pair})=\dsp{\cup_{k\in \N}}P(X=2k)$ donc $$P(X\text{ est pair})=\dsum_{k=0}^\infty P(X=2k)=\dsum_{k=0}^\infty \dfrac{\ee^{-\lambda}\lambda^{(2k)}}{(2k)!}=\ee^{-\lambda}\ch (\lambda)=\dfrac{1+\ee^{-2\lambda}}{2}.$$
  4. $P(XY \text{ est pair})=P(X\text{ est pair})P(Y=1)+P(X \geq 1)P(Y=2)$, donc $$P(XY \text{ est pair})=\dfrac{1+\ee^{-2\lambda}}{2}\dfrac{1}{2}+1\times\dfrac{1}{2} =\dfrac{3+\ee^{-2\lambda}}{4}.$$

($\star$) Correction

On considère un dé cubique, dont les faces sont numérotées de $1$ à $6$, truqué de sorte que la probabilité d'obtenir la face numéro $k$ soit proportionnelle à $k$ . On lance une fois le dé et on note $X$ le numéro de la face obtenue.

  1. Déterminer la loi de $X$, puis calculer $E(X)$.
  2. On pose $Y =\frac1X$. Déterminer la loi de $Y$ et $E(Y)$.

Correction

  1. Puisque la probabilité d'obtenir la face numéro $k$ est proportionnelle à $k$, alors en notant $\alpha=P(X=1)$, donc pour $k\in \inter{1,6},\,\,P(X=k)=\alpha k$, il faut maintenant déterminer la valeur de $\alpha$, $$1=\dsum_{k=1}^6P(X=k)=\alpha \dsum_{k=1}^6 k=21\alpha \Longrightarrow \alpha=\dfrac{1}{21}.$$ Puis, $E(x)=\dsum_{k=1}^6kP(X=k)=\dfrac{1}{21}\dsum_{k=1}^6k^2=\dfrac{13}{3}$.
  2. Puisque $X(\Omega)=\inter{1,6}$ alors $Y(\Omega)=\{\frac{1}{k},\,1\leq k\leq 6\}$, et $$P(Y=\frac{1}{k})=P(X=k)=\dfrac{k}{21},\,\, E(Y)=\dsum_{k=1}^6\dfrac{1}{k}P(Y=\frac{1}{k})=\dfrac{2}{7}.$$

(CCP PSI 2017) Correction

Déterminer la loi d'une VA $X$ telle que: $$X(\Omega)=\N^*,\quad \exists k\in ]0,1[,\,\forall n\in \N^*,\quad P(X=n)=kP(X\geq n).$$

Correction

D'après la condition vérifiée par $X$, on a $$ P(X=1)=kP(X\geq 1)=k\dsum_{j=1}^\infty P(X=j)=k,\quad \left(\text{ car } (X\geq 1)=\bigcup_{j\geq 1} (X=j)\right). $$ Essayons de calculer $P(X=2)$, $$P(X=2)=kP(X\geq 2)=k\dsum_{j\geq 2}P(X=j)=k\left(\dsum_{j\geq 1}P(X=j)-P(X=1)\right)=k(1-k).$$ Maintenant, calculons $P(X=3)$, $$ \begin{array}{lcl} P(X=3)&=&kP(X\geq 3)=k\dsum_{j\geq 3}P(X=j)\\ &=&k\left(\dsum_{j\geq 1}P(X=j)-P(X=1)-P(X=2)\right)\\ &=&k\left(1-k-k(1-k)\right)=k(1-k)^2. \end{array}$$ On peut alors mettre l'hypothèse suivante: Pour tout $n\in \N^*$, $P(X=n)=k(1-k)^{n-1}$.
Pour $n=1$ l'hypothèse est vérifiée. Soit $n\in \N^*$, supposons que l'hypothèse est vraie pour tout $k\in \inter{1,n}$ et essayons de la montrer pour $n+1$, $$\begin{array}{lcl} P(X=n+1)&=&kP(X\geq n+1)= k\dsum_{j=n+1}^\infty P(X=j)\\ &=&k\left(\dsum_{j=1}^\infty P(X=j)-\dsum_{j=1}^nP(X=j)\right) =k\left(1-\dsum_{j=1}^n k(1-k)^{j-1}\right)\\ &=&k\left(1-k\dfrac{1-(1-k)^n}{1-(1-k)}\right)=k(1-k)^n \end{array}$$ On en déduit que la relation est vérifiée au rang $n+1$, ce qui termine la démonstration.

Moments

($\heartsuit\heartsuit\heartsuit$) Correction

Soit $X$ une variable aléatoire à valeurs dans $\mathbb{N}$. Montrer que $X$ admet une espérance finie si, et seulement si, la série $\dsum {P(X > n)} $ converge et qu'alors $E\left( X \right) = \dsum\limits_{n = 0}^{ + \infty } {P\left( {X > n} \right)} $.

Correction

Soit $N\geq 1$, on a $$\begin{array}{lcl} \dsum_{k=0}^NkP(X=k)&=&\dsum_{k=1}^Nk\left(P(X>k-1)-P(X>k )\right)=\dsum_{k=1}^NkP(X>k-1)-\dsum_{k=0}^NkP(X>k)\\ &=&\dsum_{k=0}^{N-1}(k+1)P(X> k)-\dsum_{k=0}^{N}kP(X> k)=-NP(X> N)+\dsum_{k=0}^{N-1}P(X>k). \end{array}$$ On en déduit, $$\forall N\geq 1,\quad \dsum_{k=0}^{N}P(X\geq k)-\dsum_{k=0}^NkP(X=k)=(N-1)P(X>N)=(N-1)\dsum_{k=N+1}^\infty P(X=k).$$ Supposons que $E(X)$ est fini, alors $(N-1)\dsum_{k=N+1}^\infty P(X=k)\leq \dsum_{k=N+1}^\infty kP(X=k)\tendvers{N}{\infty}0$, donc $$\limiteX{N}{\infty}\left(\dsum_{k=0}^{N}P(X> k)-\dsum_{k=0}^NkP(X=k)\right)=0\Longrightarrow \dsum_{k\geq 0}P(X< k)\text{ converge et, }\dsum_{k\geq 0}P(X> k)=\dsum_{k\geq 0}kP(X=k)=E(X).$$ Inversement, supposons que $\dsum_{k\geq 0}P(X> k)$ converge, d'après la relation ci-dessus, on a $$\forall N\geq 1,\,\, 0\leq \dsum_{k=0}^NkP(X=k)\leq \dsum_{k=0}^{N}P(X> k)\Longrightarrow \dsum_{k\geq 0} kP(X=k)\text{ converge}.$$

($\star\star$) Correction

Une urne $U$ contient des boules numérotées $1,2,\,3$; une urne $V$ contient des boules numérotées $4,5,\,6$. On lance un dé. On change alors d'urne la boule dont le numéro a été tiré, on note $X_n$ le nombre de boules dans l'urne $U$. Exprimer $P(X_{n+1}=k)$ en fonction de tous les $P(X_{n+1}=j)$ possibles. En déduire que les espérances vérifiant $E(X_{n+1})=\frac{2}{3}E(x_n)+1$. Calculer $E(X_n)$.

Correction

Pour tout $n\in \N$, la variable $X_n$ prend pour valeurs $j\in \inter{0,6}$. Ainsi $\left(X_n=j\right)_{j\in \inter{0,6}}$ forment un système complet d'événements. En utilisant la formule de probabilités totale, on trouve: $$\forall n\in \N,\,\forall k\in \inter{0,6},\quad P\left(X_{n+1}=k\right)=\dsum_{j=0}^6P\left(X_n=j\right)P_{(X_n=j)}\left(X_{n+1}=k\right),$$ Or si $j\neq k-1$ ou $j\neq k+1$, alors $P_{(X_n=j)}\left(X_{n+1}=k\right)=0$.
Notons $A$ l'événement "Le numéro tiré est le numéro d'une boule de $U$", alors $$P_{(X_n=k+1)}\left(X_{n+1}=k\right)=P_{(X_n=k+1)}(A)=\dfrac{k+1}{6},\text{ et }P_{(X_n=k-1)}\left(X_{n+1}=k\right)=P_{(X_n=k-1)}\left(\overline{A}\right)=\dfrac{6-k+1}{6}.$$ On obtient alors, $$P\left(X_{n+1}=0\right)=\dfrac{1}{6}P\left(X_{n}=1\right),\,P\left(X_{n+1}=6\right)=\dfrac{1}{6}P\left(X_{n}=5\right), $$ et $$\forall k\in \inter{2,5},\,P\left(X_{n+1}=k\right)=\dfrac{6-k+1}{6}P\left(X_{n}=k-1\right)+\dfrac{k+1}{6}P\left(X_n=k+1\right).$$ Ce qui donne $$\begin{array}{lcl} E\left(X_{n+1}\right)&=&\dsum_{k=0}^6kP\left(X_{n+1}=k\right)=\dsum_{k=1}^5k\left(\dfrac{6-k+1}{6}P\left(X_{n}=k-1\right)+\dfrac{k+1}{6}P\left(X_n=k+1\right)\right)+P(X_n=5)\\ &=&\dsum_{k=1}^6 k\frac{6-k+1}{6}P(X_n=k-1)+\dsum_{k=0}^5 k\frac{k+1}{6}P(X_n=k+1)\\ &=&\dfrac{1}{6}\left(\dsum_{k=0}^64kP(X_n=k)+6\dsum_{k=0}^6P(X_n=k)\right)=\dfrac{2}{3}E(X_n)+1 \end{array}$$ On en déduit alors, que $E(X_n)=\left(\frac{2}{3}\right)^n\left(E(X_0)-3\right)+3$, comme $E(X_0)=3$ alors $E(X_n)=3$.

($\star\star$) Correction

Une urne contient au départ une boule verte et une boule rouge. On fait des tirages successifs avec la règle suivante: Quand une boule rouge sort, on s'arrête, quand une boule verte sort on la remet et on rajoute une boule rouge.
On note $X$ le nombre de tirage exécutés au total, et $V_k$ l'événement : "une boule verte est tirée au tirage n$^{\circ}\,k$"".

  1. Déterminer, pour $n\geq 2$, $P_{V_1\cap\cdots \cap V_{n-1}}(V_n)$, puis $P(X>n)$.
  2. Déterminer, pour tout $k\in \N^*,\,\,P(X=k)$. $X$ est-elle une variable aléatoire?
  3. Montrer que $X$ admet une espérance, puis la calculer.

Correction

  1. Soit $n\geq 2$, on a $$P(V_1\cap\cdots \cap V_{n-1})=\dfrac{1}{2}\dfrac{1}{3}\cdots \dfrac{1}{n}=\dfrac{1}{n!}$$
  2. Il est clair que $P(X=1)=\dfrac{1}{2}$, pour $n\geq 2$, on a $$P(X=n)=P(V_1\cap\cdots \cap V_{n-1} \cap \overline{V_n})=\dfrac{n-1}{n!}$$ D'autre part la série $\dsum \dfrac{n-1}{n!}$ converge et on a $$\dsum_{n=1}^\infty\dfrac{n-1}{n!}=\dsum_{n=1}^\infty\dfrac{1}{(n-1)!}-\dsum_{n=1}^\infty\dfrac{1}{n!}=1$$ donc $X$ est une VAD.
  3. La série $\dsum kP(X=k)$ est une série convergente, puisque $kP(X=k)=\mathrm{O}(\frac{1}{k^2})$, et on a $$E(X)=\dsum_{n\geq 1}nP(X=n)=\dsum_{n\geq 1}\dfrac{n (n-1)}{n!}=\dsum_{n\geq 2}\dfrac{1}{(n-2)!}=\ee-1.$$

($\star\star$) Correction

Soit $X$ une variable aléatoire à valeurs dans $\mathbb{N}$ telle qu'il existe $a \in \mathbb{R}$ et $p \in \left] {0,1} \right[$ vérifiant: $$P(X = k) = a\left( \begin{gathered} n + k \\ k \\ \end{gathered} \right)p^k .$$ Calculer l'espérance et la variance de $X$.

Correction

Pour $x\in ]-1,1[$, on pose $f(x)=\dsum_{k\geq 0}x^k=\dfrac{1}{1-x}$, on sait que $f\in \CC^\infty(]-1,1[)$, de plus, $$f'(x)=\dsum_{k\geq 0}(k+1)x^k,\quad f''(x)=\dsum_{k\geq 0}(k+2)(k+1)x^k,\quad f^{(3)}(x)=\dsum_{k\geq 0}(k+3)(k+2)(k+1)x^k,\cdots$$ Ainsi, on peut montrer par récurrence que, pour tout $n\in \N$, pour tout $x\in ]-1,1[,\,\,f^{(n)}(x)=n!\dsum_{k\geq 0}{n+k\choose k}x^k$.
$X$ est une variable aléatoire, alors, $$1=\dsum_{k\geq 0}P(X=k)=a\dsum_{k\geq 0}{n+k\choose k}p^k=\dfrac{a}{n!}f^{(n)}(p)=\dfrac{a}{n!}\dfrac{n!}{(1-p)^{n+1}}\Longrightarrow a=(1-p)^{n+1}.$$ On trouve également que $g_X(t)=\dfrac{n(1-p)^{n+1}}{(1-pt)^{n+1}}$, donc $g_X$ est de classe $\CC^\infty$ au voisinage de $1$ (puisque $p< 1$) donc $X$ admet une espérance et une variance. $$\begin{array}{lcl} E(X)&=&\dsum_{k\geq 0}kP(X=k)\\ &=&\dfrac{a}{n!}\dsum_{k\geq 1}(k+n)\cdots (k+1)k p^k\\ &=&\dfrac{a}{n!}\dsum_{k\geq 0}(k+n+1)\cdots (k+2)(k+1) p^{k+1}\\ &=&\dfrac{ap (n+1)!}{n!}f^{(n+1)}(p)=\dfrac{(n+1)p}{(1-p)}. \end{array} $$ Calculons maintenant $E(X^2)=\dsum_{k\geq 0}k^2P(X=k)$, $$\begin{array}{lcl} E(X^2)&=&\dfrac{a}{n!}\dsum_{k=0}^\infty (k+n+1)\cdots (k+2)(k+1)^2 p^{k+1}\\ &=&\dfrac{a}{n!}\dsum_{k=0}^\infty (k+n+1)\cdots (k+2)(k+1)k p^{k+1}+\dfrac{a}{n!}\dsum_{k=0}^\infty (k+n+1)\cdots (k+2)(k+1) p^{k+1}\\ &=&\dfrac{a}{n!}\dsum_{k=0}^\infty (k+n+2)\cdots (k+3)(k+2)(k+1) p^{k+2}+ \dfrac{a}{n!}pf^{(n+1)}(p)\\ &=&\dfrac{ap^2}{n!}f^{(n+2)}(p)+\dfrac{ap}{n!}f^{(n+1)}(p)\\ &=&\dfrac{p^2 (n+2)(n+1)}{(1-p)^2}+\dfrac{p(n+1)}{(1-p)}=\dfrac{(n+1)^2p^2+(n+1)p}{(1-p)^2} \end{array}$$ On en déduit alors, $$V(x)=E(X^2)-E(X)^2=\dfrac{(n+1)^2p^2+(n+1)p}{(1-p)^2}-\dfrac{(n+1)^2p^2}{(1-p)^2}=\dfrac{(n+1)p}{(1-p)^2}.$$

($\star$) Correction

Soit $X$ une variable aléatoire suivant une loi géométrique de paramètre $p$. Calculer $E\left( {\frac{1}{X}} \right)$.

Correction

Rappelons que, pour tout $x\in ]-1,1[$, on a $$\dfrac{1}{1-x}=\dsum_{k=0}^\infty x^k,\quad -\ln(1-x)=\dsum_{k=0}^\infty \dfrac{x^{k+1}}{k+1}=\dsum_{k\geq 1}\dfrac{x^k}{k}.$$ En utilisant la formule de transfert, on a $$E\left(\frac{1}{X}\right)=\dsum_{k\geq 1}\dfrac{1}{k}p(1-p)^{k-1}=\dfrac{p}{(1-p)}\dsum_{k\geq 1}\dfrac{(1-p)^k}{k}=\dfrac{-p\ln(1-(1-p))}{1-p}=\dfrac{-p\ln(p)}{1-p}.$$

(Classique $\star\star$) Correction

Soit $X$ une variable aléatoire à valeurs dans~$\N$.

  1. Montrer que $X$ a une espérance si et seulement si la série $\dsum_{k\geq 1}P(X\geq k)$ est convergente et que dans ce cas, $E(X)$ est la somme de cette série.
  2. Établir une formule analogue pour $V(X)$ en fonction de $\dsum_{k\geq 1}P(X\geq k)$ et $\dsum_{k\geq 1}kP(X\geq k)$.

Correction

  1. Soit $N\geq 1$, on a $$\begin{array}{lcl} \dsum_{k=0}^NkP(X=k)&=&\dsum_{k=0}^Nk\left(P(X\geq k)-P(X\geq k+1)\right)=\dsum_{k=0}^NkP(X\geq k)-\dsum_{k=0}^NkP(X\geq k+1)\\ &=&\dsum_{k=0}^{N}kP(X\geq k)-\dsum_{k=1}^{N+1}(k-1)P(X\geq k)=-NP(X\geq N+1)+\dsum_{k=1}^{N}P(X\geq k). \end{array}$$ On en déduit, $$\forall N\geq 1,\quad \dsum_{k=1}^{N}P(X\geq k)-\dsum_{k=0}^NkP(X=k)=NP(X\geq N+1)=N\dsum_{k=N+1}^\infty P(X=k).$$ Supposons que $E(X)$ est fini, alors $N\dsum_{k=N+1}^\infty P(X=k)\leq \dsum_{k=N+1}^\infty kP(X=k)\tendvers{N}{\infty}0$, donc $$\limiteX{N}{\infty}\left(\dsum_{k=1}^{N}P(X\geq k)-\dsum_{k=0}^NkP(X=k)\right)=0\Longrightarrow \dsum_{k\geq 1}P(X\geq k)\text{ converge et, }\dsum_{k\geq 1}P(X\geq k)=\dsum_{k\geq 0}kP(X=k)=E(X).$$ Inversement, supposons que $\dsum_{k\geq 1}P(X\geq k)$ converge, d'après la relation ci-dessus, on a $$\forall N\geq 1,\, 0\leq \dsum_{k=0}^NkP(X=k)\leq \dsum_{k=1}^{N}P(X\geq k)\Longrightarrow \dsum_{k\geq 0} kP(X=k)\text{ converge}.$$
  2. Soit $N\geq 1$, on a $$\dsum_{k=1}^N 2kP(X\geq k)-\dsum_{k=0}^{N-1}k^2P(X=k)=\dsum_{k=1}^{N-1}P(X\geq k)+(N^2+1)P(X\geq N),$$ donc en supposant que $E(X)$ est finie, on obtient que $E(X^2)$ est finie si et seulement si la série $\dsum_{k=1}^\infty kP(X\geq k)$ converge et dans ce cas, $$V(X) = 2\dsum_{k=1}^\infty kP(X\geq k)-\dsum_{k=1}^\infty P(X\geq k) - \Bigl(\dsum_{k=1}^\infty P(X\geq k)\Bigr)^2.$$

($\star$) Correction

Soit $X$ une VAD suivant la loi de Poisson de paramètre $\lambda>0$. Calculer $E\left(\frac{1}{1+X}\right)$.

Correction

$$\boxed{E\left(\frac{1}{1+X}\right)=\dfrac{1-\ee^{-\lambda}}{\lambda}}.$$

(Oral PSI 2017) Correction

A quelle condition sur $r$ peut-on définir une VA $X$ telle que $X(\Omega)=\N$ et $P(X=n)=\dfrac{(2n)!r}{2^{3n}(n!)^2}$. Montrer que, quand cette condition est réalisée, $X$ admet une espérance et une variance et les calculer.

Correction

On pose $f(x)=\dsum_{n\geq 0}\dfrac{(2n)!r}{2^{3n}(n!)^2}x^n$, en utilisant le règle de D'Almbert, on trouve que le rayon de convergence de cette SE est $2$, donc $f\in \CC^\infty(]-2,2[)$.
Pour tout $x\in ]-1,1[$, on a $$\begin{array}{lcl} f'(x)&=&\dsum_{n\geq 1}\dfrac{(2n)!r n}{2^{3n}(n!)^2}x^{n-1}\\ &=&\dsum_{n\geq 0}\dfrac{(2n+2)!r }{2^{3n+3}(n!)(n+1)!}x^{n}=\dsum_{n\geq 0}\dfrac{(2n+1)(2n)!r}{4 2^{3n}(n!)^2}x^n\\ &=&\dfrac{x}{2}f'(x)+\dfrac{1}{4}f(x) \end{array} $$ Ainsi $f$ est l'unique solution du problème de Cauchy suivant, $$(y(0)=r,\quad (1-\frac{x}{2})y'-\frac{1}{4}y=0,$$ la résolution de ce problème nous donne $$\forall x\in ]-2,2[,\quad f(x)=\dfrac{r}{\sqrt{1-\frac{x}{2}}}.$$ On en déduit alors que la condition sur $r$ est $r=\dfrac{1}{\sqrt{2}}$, puisque $f(1)=\sqrt{2}r=1$.
Ainsi, $G_X(t)=\dfrac{1}{\sqrt{2}}\dfrac{1}{\sqrt{1-\frac{t}{2}}}$, comme $R_c>1$ alors $X$ admet une espérance et une variance.

(PC 2017) Correction

On donne $X$ une VA telle que $X(\Omega)=\N^*$ et $P(X=k)=\dfrac{\alpha}{k(k+1)(k+2)}$.

  1. Déterminer la valeur de $\alpha$.
  2. $X$ admet une espérance? une variance? Si oui, la (les ) calculer.

Correction

  1. D'abord il faut que $\alpha$ soit $\geq 0$, car $P(X=k)\geq 0$, ensuite il faut que $\dsum_{k=1}^\infty P(X=k)=1$.
    Pour calculer cette somme, on écrit, $$\forall k\geq 1,\,\dfrac{1}{k(k+1)(k+2)}=\dfrac{A}{k}+\dfrac{B}{k+1}+\dfrac{C}{k+2}=\dfrac{1/2}{k}+\dfrac{-1}{k+1}+\dfrac{1/2}{k+2},$$ Soit $N\geq 1$, on a $$\begin{array}{lcl} \dsum_{k=1}^N \dfrac{a}{k(k+1)(k+2)}&=&\dsum_{k=1}^N \left(\dfrac{a/2}{k}+\dfrac{-a}{k+1}+\dfrac{a/2}{k+2}\right)=\dfrac{a}{2} \dsum_{k=1}^N \left(\dfrac{1}{k}-\dfrac{1}{k+1}\right)-\dfrac{a}{2}\dsum_{k=1}^N \left(\dfrac{1}{k+1}-\dfrac{1}{k+2}\right)\\ &=&\dfrac{a}{2}\left(1-\dfrac{1}{N+1}\right)-\dfrac{a}{2}\left(\dfrac{1}{2}-\dfrac{1}{N+2}\right)\tendvers{N}{\infty}\dfrac{a}{4}\Longrightarrow\boxed{ a=4}. \end{array}$$
  2. $kP(X=k)\underset{k\to \infty}{\sim}\dfrac{4}{k^2}$, comme $\dsum\dfrac{1}{k^2}$ converge, on en déduit que $\dsum kP(X=k)$ converge, donc $X$ admet une espérance.\\ De plus, pour tout $N\geq 1$, on a $$\begin{array}{lcl} \dsum_{k=1}^N kP(X=k)&=&\dsum_{k=1}^N \left(\dfrac{4}{(k+1)(k+2)}\right)=4 \dsum_{k=1}^N \left(\dfrac{1}{k+1}-\dfrac{1}{k+2}\right)\\ &=&4\left(\dfrac{1}{2}-\dfrac{1}{N+2}\right)\tendvers{N}{\infty}2\Longrightarrow\boxed{ E(X)=2}. \end{array}$$ $k^2P(X=k)\underset{k\to \infty}{\sim}\dfrac{4}{k}$, comme $\dsum\dfrac{1}{k}$ diverge, on en déduit que $\dsum k^2P(X=k)$ diverge, donc $X$ n'admet pas une variance.

(Mines PSI 2017) Correction

On pioche une poignée de jetons dans une urne en contenant $n$, numérotés de $1$ à $n$; on admet que chaque poignée (y compris la poignée vide) a la même probabilité d'être tirée. Donner l'espérance de la variable aléatoire S donnant la somme des numéros tirés.

Correction

Supposons que $n\geq 1$ et notons $N=\dfrac{n(n+1)}{2}$, il est clair que $S(\Omega)=\inter{0,N}$.

Lorsque on tire une poignée de longueur $k$ ($k\in \inter{0,n}$) et de somme $\alpha$, il nous reste dans l'urne $n-k$ jetons dont la somme est $N-\alpha$. On en déduit que, $$\forall m\in \inter{0,N},\quad P(S=m)=P(S=N-m).$$ Supposons maintenant que $N=2M+1$, alors $$\begin{array}{lcl} E(S)&=&\dsum_{m=0}^NmP(S=m)\\ &=&\dsum_{m=0}^M mP(S=m)+\dsum_{m=M+1}^{2M+1} mP(S=m)\\ &=&\dsum_{m=0}^M mP(S=m)+\dsum_{m=0}^{M} (N-m)P(S=N-m)=\dsum_{m=0}^M NP(S=m) =N/2 \end{array} $$ En effet, on a $$1=\dsum_{m=0}^NP(S=m)=\dsum_{m=0}^MP(S=m)+\dsum_{m=M+1}^NP(S=m)=2\dsum_{m=0}^M$$ Si $N=2M$, alors $$\begin{array}{lcl} E(S)&=&\dsum_{m=0}^NmP(S=m)\\ &=&\dsum_{m=0}^{M-1} mP(S=m)+MP(S=M)+\dsum_{m=M+1}^{2M} mP(S=m)\\ &=&\dsum_{m=0}^{M-1} mP(S=m)+MP(S=M)+\dsum_{m=0}^{M-1} (N-m)P(S=N-m)=\dsum_{m=0}^M NP(S=m) =N/2 \end{array} $$

Fonctions génératrice

($\star$) Correction

Soient $X\hookrightarrow \mathcal{P}(a),Y\hookrightarrow \mathcal{P}(a)$ deux v.a.d indépendantes ($a\in \R$). On pose $Z=X+3Y$, déterminer la fonction génératrice de $Z$ puis trouver, de deux manières différentes, l'espérance et la variance de $Z$.

Correction

$X$ suit une loi de Poisson de paramètre $a$ donc $G_X$ est définie sur $\R$ et on a $G_X(t)=\ee^{a(t-1)}$.
Il faut déterminer la fonction génératrice de $3Y$, vu que $P(3Y=n)=0$ si $n$ n'est pas un multiple de $3$, alors $$g_{3Y}(t)=\dsum_{k=0}^\infty P(3Y=k)t^k=\dsum_{k\in \N}P(3Y=3k)t^{3k}=\dsum_{k\in \N}P(Y=k)t^{3k}=g_{Y}(t^3)=\ee^{a(t^3-1)}.$$ Comme $X,\,Y$ sont indépendantes, alors $X,\,3Y$ le sont également, on en déduit que la fonction génératrice de $Z$ est le produit des deux fonctions génératrices de $X$ et $3Y$ (qui ont le même rayon de convergence), donc $$\forall t\in \R,\,g_Z(t)=g_X(t)g_{3Y}(t)=g_X(t)g_Y(t^3)=\ee^{a(t^3+t-2)}.$$ Pour déterminer l'espérance et la variance de $Z$, on a $$E(Z)=E(X+3Y)=E(X)+3E(Y)=4a,\quad V(Z)=V(X+3Y)=V(X)+9V(Y)=10a.$$ La deuxième méthode consiste à utiliser la fonction génératrice, en effet, comme $g_Z$ est de classe $\CC^\infty$ sur $\R$, alors, $$E(Z)=g_Z'(1),\,V(Z)=g_Z''(1)+g_Z'(1)-g_Z'(1)^2.$$ Or, $$\forall t\in \R,\,g_Z'(t)=a(3t^2+1)\ee^{a(t^3+t-2)},\,g_Z''(t)=(6at+a^2(t^2+1)^2)\ee^{a(t^3+t-2)},\Longrightarrow g_Z'(1)=4a,\,g_Z''(1)=16a^2+4a.$$ En remplaçant dans les formules ci-dessus, on trouve $E(Z)=4a,\,\,V(Z)=16a^2+6a+4a-(4a)^2=10a.$

($\star$) Correction

Trouver la constante $k$ et une condition sur $a\in \R$ pour que : $P(X=n)=\left(\frac{a}{1+a}\right)^nk,$ définisse une variable aléatoire à valeur dans $\N$.
Pour toute suite de variables indépendantes $(X_n)$ suivant toutes la loi précédente, déterminer la fonction génératrice de $S_n=\dsum_{k=1}^nX_k$, puis trouver de deux manières différentes l'espérance et la variance de $S_n$.

Correction

$a>0$ et $k=\dfrac{1}{a+1}$, de plus, on a $$\forall t\in \R, \abs{\frac{at}{a+1}}< 1,\quad g_X(t)= \dfrac{1}{a+1}\dsum_{n\in \N}\left(\dfrac{a}{a+1}\right)^nt^n=\dfrac{1}{a+1}\dfrac{1}{1-\frac{at}{a+1}}=\dfrac{1}{a+1-at}.$$

  1. En utilisant la linéarité de l'espérance, on trouve $$ E(S_n)=E\left(\dsum_{k=1}^nX_n\right)=\dsum_{k=1}^nE(X_n)=nE(X).$$ puis comme le rayon de convergence de $R_X=\dfrac{a+1}{a}>1$, alors $$E(X)=G_X'(1),\, G_X'(t)=\dfrac{a}{(1+a-at)^2}\Longrightarrow E(X)=a$$ donc $E(S_n)=na$.
  2. En utilisant la fonction génératrice de $S_n$, puisque les $X_k$ sont indépendantes, alors $$G_{S_n}(t)=\dsp\prod_{k=1}^n G_{X_k}(t)= G_X(t)^n.$$ donc $G'_{S_n}(t)=nG'_X(t)G_X(t)^{n-1}$, en particulier, $G'_{S_n}(1)=nG'_X(1)G_X(1)^{n-1}=na$, on retrouve donc $E(S_n)=na$.
($\star\star$) Correction

Soit $p\in ]0,1[$. Déterminer $c$ pour que $P(X=n)=\dfrac{p^{n+1}}{c(n+1)}$ définissent la loi de probabilité d'une VAD $X$.
Déterminer la fonction génératrice de $X$, son espérance et sa variance.

Correction

Il faut que $\dsum_{n\geq 0}P(X=n)$ soit égale à $1$, or $$\dsum_{n\geq 0}P(X=n)=\dsum_{n\geq 0}\dfrac{p^{n+1}}{c(n+1)}=\dfrac{-\ln(1-p)}{c}\Longrightarrow c=-\ln(1-p).$$ Soit $t\in \R$ tel que $\abs{t}\leq \frac{1}{p}$, on a $$g_X(t)=\dsum_{k\geq 0}P(X=k)t^k=\dfrac{1}{-\ln(1-p)}\dsum_{k\geq 0}\dfrac{p^{k+1}t^k}{k+1}=\left\{\begin{array}{lcl} \dfrac{\ln(1-pt)}{t\ln(1-p)}&\text{ si }& t\neq 0\\ &&\\ \dfrac{p}{-\ln(1-p)}&\text{ si }& t=0 \end{array} \right.$$ La fonction génératrice a un rayon de convergence $>1$ (puisque $p< 1$), donc $g_X$ est de classe $\CC^\infty$ au voisinage de $1$. Soit $t$ au voisinage de $1$, on a $$g_X'(t)=\dfrac{1}{\ln(1-p)}\dfrac{-pt-(1-pt)\ln(1-pt)}{t^2(1-pt)}$$ donc $$E(X)=g_X'(1)=\dfrac{1}{\ln(1-p)}\dfrac{-p-(1-p)\ln(1-p)}{(1-p)}=-1-\dfrac{p}{(1-p)\ln(1-p)}.$$ Ensuite, $$g_X''(t)=\dfrac{p\ln(1-pt)(t^2(1-pt))+(pt+(1-pt)\ln(1-pt))(2t-3pt^2)}{\ln(1-p)t^4(1-pt)^2},$$ soit $$g_X''(1)=\dfrac{p\ln(1-p)(1-p)+(p+(1-p)\ln(1-p))(2-3p)}{\ln(1-p)(1-p)^2}.$$ Ce qui donne $$\begin{array}{lcl} V(X)&=&g_X''(1)+g_X'(1)-(g_X'(1))^2\\ &=&\dfrac{p\ln(1-p)(1-p)+(p+(1-p)\ln(1-p))(2-3p)}{\ln(1-p)(1-p)^2}+\left(-1-\dfrac{p}{(1-p)\ln(1-p)}\right)-\left(-1-\dfrac{p}{(1-p)\ln(1-p)}\right)^2\\ &=&\dfrac{(2-2p)(1-p)\ln(1-p)+p(2-3p)}{(1-p)^2\ln(1-p)}-2-\dfrac{3p}{(1-p)\ln(1-p)}-\dfrac{p^2}{(1-p)^2\ln(1-p)^2}\\ &=&\dfrac{-p}{(1-p)^2\ln(1-p)}-\dfrac{p^2}{(1-p)^2\ln(1-p)^2}=\dfrac{-p}{(1-p)^2\ln(1-p)}\left(1+\dfrac{p}{\ln(1-p)}\right) \end{array}$$

($\star\star$) Correction

On tire, dans une urne contenant $a$ boules blanches et $b$ boules noires, une boule au hasard. Si la boule obtenue est blanche, on la remet, si elle est noire, on la remplace par une boule blanche.
On note $Y$ le rang de la première boule blanche obtenue lors des ces tirages successifs et $Z=b+1-Y$. Déterminer la fonction génératrice de $Z$, en déduire l'espérance de $Z$ et de $Y$.

Correction

Pour $n\in \N^*$, on note $A_n$ l'événement " La boule tirée au tirage n° $n$ est noire", Donc, pour $k\in \inter{2,b+1}$, on a $$P(Y=k)=P(A_1\cap A_2\cdots \cap A_{k-1}\cap \overline{A}_k)=\dfrac{b(b-1)\cdots (b-k+2)}{(a+b)^{k-1} }\dfrac{a+k-1}{(a+b)}$$ et $P(Y=1)=\dfrac{a}{a+b}$, d'autre part pour tout $k\geq b+2,\, P(Y=k)=0$. Pour résumé, $$Y(\Omega)=\inter{1,b+1},\,\forall k\in \inter{1,b},\, P(Y=k)= \dfrac{b!}{(b-k+1)!}\dfrac{a+k-1}{(a+b)^k}.$$ Comme $Z=b+1-Y$ alors $Z(\Omega)=\inter{0,b}$ et pour $0\leq k\leq b$ $$P(Z=k)=P(Y=b+1-k)= \dfrac{b!}{(b-(b+1-k))!}\dfrac{a+(b+1-k)-1}{(a+b)^{b+1-k}}= \dfrac{b!}{k!}\dfrac{a+b-k}{(a+b)^{b+1-k}}$$ Donc $$G_Z(t)=\dfrac{b!}{(a+b)^{b+1}}\dsum_{k=0}^b \dfrac{(a+b-k)(a+b)^k}{k!}t^k.$$ Puis, $$\forall t\in \R,\, G'_Z(t)=\dfrac{b!}{(a+b)^{b+1}}\dsum_{k=0}^b \dfrac{k(a+b-k)(a+b)^k}{k!}t^{k-1} $$ En particulier, $$\begin{array}{lcl} G'_Z(1)&=& \dfrac{b!}{(a+b)^{b+1}}\dsum_{k=0}^b \dfrac{k(a+b-k)(a+b)^k}{k!}\\ &=&\dfrac{b!}{(a+b)^{b+1}}\dsum_{k=0}^{b-1} \dfrac{(a+b-k-1)(a+b)^{k+1}}{k!}\\ &=&\dfrac{b!}{(a+b)^{b+1}}\dsum_{k=0}^{b-1} \dfrac{(a+b-k)(a+b)^{k+1}}{k!}-\dfrac{b!}{(a+b)^{b+1}}\dsum_{k=0}^{b-1} \dfrac{(a+b)^{k+1}}{k!} \end{array} $$ or, $$\begin{array}{lcl} \dsum_{k=0}^{b-1} \dfrac{(a+b-k)(a+b)^{k+1}}{k!}&=&\dsum_{k=0}^{b-1} \dfrac{(a+b)(a+b)^{k+1}}{k!}-\dsum_{k=0}^{b-1} \dfrac{k(a+b)^{k+1}}{k!}\\ &=&\dsum_{k=0}^{b-1} \dfrac{(a+b)^{k+2}}{k!}-\dsum_{k=1}^{b-1} \dfrac{(a+b)^{k+1}}{(k-1)!}\\ &=&\dsum_{k=0}^{b-1} \dfrac{(a+b)^{k+2}}{k!}-\dsum_{k=0}^{b-2} \dfrac{(a+b)^{k+2}}{k!}\\ &=&\dfrac{(a+b)^{b+1}}{(b-1)!} \end{array}$$ Ce qui donne $$G'_Z(1)=b-\dfrac{b!}{(a+b)^{b+1}}\dsum_{k=0}^{b-1} \dfrac{(a+b)^{k+1}}{k!}=b-\dfrac{b!}{(a+b)^{b}}\dsum_{k=0}^{b-1} \dfrac{(a+b)^{k}}{k!}$$ On en déduit finalement, $$ E(Z)=b-\dfrac{b!}{(a+b)^{b}}\dsum_{k=0}^{b-1} \dfrac{(a+b)^{k}}{k!}, \, E(Y)=b+1-E(Z)=1+\dfrac{b!}{(a+b)^{b}}\dsum_{k=0}^{b-1} \dfrac{(a+b)^{k}}{k!}$$ Par exemple, $$(a,b)= (3,2)\Longrightarrow E(Y)= 1.48$$ Par contre, pour $$(a,b)=(3,100)\Longrightarrow E(Y)= 10.71623563$$ ce qui est normale, il faut attendre un peu plus dans ce cas pour voir la 1ère boule blanche!

($\star\star$) Correction

Une urne contient 4 boules numérotées $0,\,1,\,1,\,2$. On effectue $n$ triages avec remise et on note $S_n$ la somme des numéros tirés.
Donner la loi de $S_n$ et déterminer l'espérance de $S_n$.

Correction

Pour $k\in \inter{1,n}$, on note $X_k$ le VAD qui donne le numéro de la $k$ème tirage. Les $(X_k)$ sont indépendantes et suivent la même loi $X$ tel que $$P(X=0)=\dfrac{1}{4},\quad P(X=1)=\dfrac{1}{2},\quad P(X=2)=\dfrac{1}{4}.$$ Il est claire que $S_n=\dsum_{k=1}^n X_k$, on en déduit que $G_{S_n}=\dsp\prod_{k=1}^nG_{X_k}=G_X^n$. Or $$\forall t\in \R,\,G_X(t)=P(X=0)t^0+P(X=1)t+P(X=2)t^2=\left(\frac{1+t}{2}\right)^2$$ Ce qui donne: $$G_{S_n}(t)=\left(\frac{1+t}{2}\right)^{2n}=\dfrac{1}{2^{2n}}\dsum_{k=0}^{2n}{2n\choose k}t^k.$$ On en déduit, $$\boxed{\forall k\in \inter{1,2n}\,\quad P(S_n=k)=\dfrac{1}{2^{2n}}{2n\choose k}.}$$

(CCP PSI 2018) Correction

Soient $X$ et $Y$ deux variables aléatoires indépendantes et de même loi. Soit $Z$ une variable aléatoire suivant une loi géométrique de paramètre $p$ telle que $Z=X+Y+1$.

  1. Trouver l'espérance et la variance de $X$.
  2. Calculer $G_X(t)$.
  3. Trouver la loi de $X$.

Correction

  1. Comme $X,Y$ suivent la même loi, alors (sous réserve d'existence) ils ont la même espérance et variance. D'autre part, $X+Y=Z-1$ a une espérance et variance finie, on en déduit que $e=\mathrm{E}(X)= \mathrm{E}(Y)< \infty$ et $v=\mathrm{V}(X)=\mathrm{V}(Y)< \infty$,
    1. En utilisant la linéarité de l'espérance, on trouve: $$2e=\mathrm{E}(Z-1)=\mathrm{E}(Z)-\mathrm{E}(1)=\dfrac{1}{p}-1\Longrightarrow\boxed{ e =\dfrac{1-p}{2p}}.$$
    2. En utilisant la définition de la variance et le faite que $X$ et $Y$ sont indépendantes, on trouve: $$2v=\mathrm{V}(Z-1)=\mathrm{V}(Z)-\mathrm{V}(1)=\dfrac{1-p}{p^2}-0\Longrightarrow\boxed{ v =\dfrac{1-p}{2p^2}}.$$
  2. Comme $X$ et $Y$ sont indépendantes, alors $G_{X+Y}(t)=G_X(t)^2$, or $$\begin{array}{lcl} G_{X+Y}(t)&=&G_{Z-1}(t)=\dsum_{k=0}^\infty P(Z-1=k)t^k\\ &=&\dsum_{k=0}^\infty P(Z=k+1)t^k=\dsum_{k=0}^\infty p(1-p)^kt^k=\dfrac{p}{1-(1-p)t}\Longrightarrow\boxed{G_X(t)=\sqrt{\dfrac{p}{1-(1-p)t}}}. \end{array}$$ Le rayon de convergence de $G_X$ est alors $\dfrac{1}{1-p}$.
  3. D'après le cours sur les séries entières, on a $\dsp \forall x\in ]-1,1[,\,\dfrac{1}{\sqrt{1-x}}=\dsum_{n=0}^\infty\dfrac{(2n)!}{2^{2n}(n!)^2}x^n$, donc $$\forall x\in ]-1/(1-p),1/(1-p)[,\,\dfrac{\sqrt{p}}{\sqrt{1-(1-p)x}}=\dsum_{n=0}^\infty\dfrac{\sqrt{p}(2n)!}{2^{2n}(n!)^2}(1-p)^nx^n.$$ On en déduit, $\boxed{P(X=n)=\dfrac{\sqrt{p}(2n)!(1-p)^n}{2^{2n}(n!)^2}}$.

(Mines-Télécom PSI 2018) Correction

Soient $X$ et $Y$ deux variables aléatoires indépendantes sur un même espace probabilisé ; $X$ suit une loi de Bernoulli de paramètre $p$ et $Y$ une loi de Poisson de paramètre $a$. Soit $Z=XY$.

  1. Déterminer la loi de $Z$.
  2. Déterminer l'espérance et la variance de $Z$.
  3. Déterminer la fonction génératrice de $Z$.

Correction

  1. Soit $k\in \N$, on distingue deux cas,
    1. Si $k=0$, alors $(Z=0)=(X=0\text{ et } Y\in \N)\cup (X=1\cap Y=0)$ donc $$P(Z=0)=(1-p)\times 1+ p\times \ee^{-a}\dfrac{a^0}{0!}=(1-p)+p\ee^{-a}.$$
    2. Si $k>0$, alors $(Z=k)= (X=1\cap Y=k)$ ce qui donne $P(Z=k)=p\ee^{-a}\dfrac{a^k}{k!}.$
  2. $Z$ admet une espérance finie car $\abs{Z}\leq Y$ et $Y$ a une espérance finie. On utilise la définition de l'espérance $$\mathrm{E}(Z)=\dsum_{k\geq 0}kP(Z=k)=\dsum_{k\geq 1}p\ee^{-a}k\dfrac{a^k}{k!}=pa\ee^{-a}\dsum_{k\geq 0}\dfrac{a^k}{k!}\Longrightarrow\boxed{\mathrm{E}(Z)=pa}.$$ Pour la variance, on calcul, $$\begin{array}{lcl} \mathrm{E}(Z^2)&=&\dsum_{k\geq 0}k^2P(Z=k)=p\ee^{-a}\dsum_{k\geq 1}k\dfrac{a^k}{(k-1)!}\\ &=&p\ee^{-a}(a +\dsum_{k\geq 2}k\dfrac{a^k}{(k-1)!})=p\ee^{-a}(a +\dsum_{k\geq 1}(k+1)\dfrac{a^{k+1}}{(k)!})\\ &=&p\ee^{-a}(a +\dsum_{k\geq 1}\dfrac{a^{k+1}}{(k-1)!}+\dsum_{k\geq 1}\dfrac{a^{k+1}}{k!})=p\ee^{-a}\left(a^2\dsum_{k=0}^\infty\dfrac{a^k}{k!}+a\dsum_{k=0}^\infty\dfrac{a^k}{k!}\right) .\\ &=&pa^2+pa. \end{array}$$ on en déduit alors, $\boxed{V(Z)=E(Z^2)-E(Z)^2=pa^2+pa-p^2a^2}$.

(ENS PSI ($\star\star$)) Correction

Soient deux dés (pipés ou non) indépendants. Montrer qu'il est impossible d'obtenir la loi uniforme pour la somme S des deux dés (c'est-à-dire $P(S=i)=\dfrac{1}{11}$ pour tout $i\in \inter{2,12}$.

Correction

Notons $X_1$ (resp. $X_2$) le v.a.d qui donne le résultat de 1ère dé (resp. de 2ème dé), et $$\forall i\in \inter{1,6},\quad P(X_1=i)=\alpha_i,\quad P(X_2=i)=\beta_i$$ Bien évidement si les dés ne sont pas pipés on aura $\alpha_i=\beta_i=\dfrac{1}{6}$.
Donc $S=X_1+X_2$, supposons que $S$ suit une loi uniforme, alors on aurait $$\forall t\in \R,\,G_S(t)=\dsum_{k=2}^{12}\dfrac{t^k}{11}=\dfrac{t^2}{11}\left(1+t+\cdots+ t^{10}\right)= G_{X_1}(t)G_{X_2}(t).$$ D'autre part, $$G_{X_1}(t)=t\left(\alpha_1+\alpha_2 t+\cdots+\alpha_6 t^5\right)= t\varphi_1(t)$$ $$G_{X_2}(t)=t\left(\beta_1+\beta_2 t+\cdots+\beta_6 t^5\right)= t\varphi_2(t)$$ avec $\varphi_1,\,\varphi_2$ deux polynômes de degrés 5 (et exactement 5---justifier!).
On en déduit $$\forall t\in \R,\, \left(1+t+\cdots+ t^{10}\right)= 11\varphi_1(t)\varphi_2(t)$$ ce qui est absurde car $t\longmapsto \left(1+t+\cdots+ t^{10}\right)$ n'admet pas de racines réelles, tandis que $\varphi_1\varphi_2$ a au moins une racine réelle.